GI/Gu

¡Supera tus tareas y exámenes ahora con Quizwiz!

A nurse is changing the colostomy bag for a client with a new colostomy. Which elements of skin care should the nurse provide during this process? Select all that apply. A. Avoiding adhesive remover or water when removing the pouch from the skin B. Drying the skin thoroughly before reapplying the pouch C. Using sterile gauze to wipe the stoma and absorb exudate D. Cleaning around the stoma with warm water and a washcloth E. Avoiding ripping the pouch off of the client's skin

Avoiding ripping the pouch off of the client's skin A client who uses a colostomy bag is at risk of skin breakdown around the ostomy site. The pouch should be removed by pushing gently down on the skin with one hand, while pulling up on the pouch with the other hand. Cleaning around the stoma with warm water and a washcloth Skin care includes inspecting the skin and stoma site carefully, washing the skin with soap and water. ensuring the skin is dry before applying a new pouch. Drying the skin thoroughly before reapplying the pouch Skin care also includes ensuring the skin is dry before applying a new pouch. If the skin is irritated, it is possible that the pouch opening is too large, excessively exposing the surrounding skin to feces which leads to skin irritation and breakdown.

A client with a peptic ulcer has been brought in to the healthcare clinic and is being assessed by the nurse for an upper GI bleed. Which of the following signs or symptoms would the nurse expect to see with this condition? Select all that apply. A. Abdominal fullness B. Melena C. Swelling in the lower legs D. Epigastric pain E. Hematemesis

B, D, E....."Epigastric pain", "Melena" and "Hematemesis" are correct. An upper GI bleed occurs in the upper portion of the gastrointestinal tract, including the area within the esophagus. Signs or symptoms associated with bleeding from this area include epigastric pain, vomiting blood, and dark blood in your stool, called melena.

A nurse is caring for a client who had a gastrectomy due to stomach cancer. The nurse is teaching the client how to avoid dumping syndrome. Which of the following statements by the client demonstrates that more education is necessary? A. "I'll focus on eating high protein and high fat foods for the time being." B. "I shouldn't ever lay down after eating." C. "I will avoid milk and products with added sugar." D "I won't drink anything with meals or an hour before meals."

B. "I shouldn't ever lay down after eating." Dumping syndrome occurs due to rapid movement of food into the intestines from the stomach. Laying down after eating is actually acceptable and encouraged because it will help to slow the progression of food from the stomach into the small intestine. Therefore this client requires further clarification of teaching regarding positioning after meals.

The nurse is caring for a client diagnosed with Crohn's disease. Which statement indicates more teaching is needed? A. "A high-calorie, high-protein diet is best" B. "I will deal with chronic constipation" C. "I am at risk for anemia and electrolyte disturbances" D. "I will have periods of remission and periods of exacerbation"

B. "I will deal with chronic constipation" A client with Crohn's disease will have frequent episodes of diarrhea, not constipation.

A pregnant client is talking to the nurse about a current gonorrhea infection. Which of the following statements by the client demonstrates an understanding of the prevention of transmission to the neonate? A. "If my baby shows signs of infection, then a dose of eye ointment will be given" B. "My baby will receive a dose of IV antibiotics after birth to treat the infection" C. "Infection risk is low for infants" D. "We will move forward with a c-section to avoid transmission to my baby"

B. "My baby will receive a dose of IV antibiotics after birth to treat the infection" Infants born to mothers with active infections receive a one-time weight-based dose of ceftriaxone.

A client is preparing to undergo anorectal manometry to determine the cause of their constipation. Which of the following elements is associated with this test? Select all that apply. A. An x-ray is performed while a client consumes a meal to determine transit time B. A radial catheter is inserted into the anus C. A catheter is pulled through part of the colon to determine sphincter tone D. A client is instructed to bear down while they expel a barium enema E. A balloon on the end of a catheter is inflated in the rectum

B. A radial catheter is inserted into the anus--Anorectal manometry is a diagnostic test that determines sphincter tone in the rectum and anus. The test involves inserting a catheter into the client's rectum, inflating a balloon on the end of the catheter, and pulling the catheter through the rectum and anus to determine sphincter tone. E. A balloon on the end of a catheter is inflated in the rectum--This is done after the catheter is inserted into the client's rectum. C. A catheter is pulled through part of the colon to determine sphincter tone--Once the catheter is inserted and the balloon is inflated, it is pulled through the rectum to determine sphincter tone.

The nurse is caring for a client with peptic ulcer disease due to H. pylori. Which drug combinations should be given along with a macrolide antibiotic? A. Flagyl and Amphogel B. Amoxicillin and Prilosec C. Penicillin and Axid D. Tetracycline and sodium bicarbonate

B. Amoxicillin and Prilosec H. pylori can be complicated to treat, because the bacteria quickly becomes resistant to antibiotics. Therefore, "triple therapy" is used. (When triple therapy fails, "quadruple therapy" is recommended.) Triple therapy consists of a macrolide antibiotic, a proton pump inhibitor, and a penicillin-related antibiotic.

A client in a long-term care facility is having elimination problems and suffers from incontinence. Which nursing intervention would most likely help to prevent skin breakdown in this client? A. Inserting a urinary catheter B. Applying emollient to the skin after the client's bath C. Assisting the client to use the bedpan every eight hours D. Providing privacy when the client must use the bathroom

B. Applying emollient to the skin after the client's bath A client who is incontinent is at risk of skin breakdown when urine and feces are in contact with the skin. The enzymes in these wastes cause the skin to soften and become macerated, which leads to skin breakdown and wounds. The nurse should bathe the client regularly and apply an emollient that works as a barrier between urine and the surface of the skin, as well as apply emollient each time the skin is cleansed after an incontinent episode.

A client has been brought to the pre-op area to have his IV started before undergoing hernia surgery. The client is extremely anxious and starts crying when the nurse walks into the room. Which intervention would be most appropriate in this situation to help with the client's anxiety? A. Give the client some sips of water B. Ask the client if there is anyone that came with him that he wants nearby C. Contact the provider for prescription sedative agents D. Ask the client to calm down so the nurse can talk to him

B. Ask the client if there is anyone that came with him that he wants nearby Many clients are very nervous about having surgery, or about being in the hospital in general. The nurse can best help by making the transition into surgery as smooth as possible. Because the client in this situation seemed to become more upset at the sight of the nurse, a better choice may be to have a family member or a person close to the client stay with him until he needs to go back for surgery.

A nurse on the medical-surgical unit has been assigned to a client who is legally blind and recovering from abdominal surgery. Which interventions would the nurse implement when caring for this client in order to best uphold the client's safety while in the hospital? Select all that apply. A. Perform tasks for the client to minimize the risk of injury B. Ask the client what he is able to see C. Keep the client's belongings in the same place and do not move them unnecessarily D. Place an identifier sign above the client's bed notifying staff of the client's vision loss E. Remind the client to call for help whenever getting out of bed

B. Ask the client what he is able to see--A client with vision loss may need some assistance while in the hospital and the nurse would need to take measures to uphold the client's safety. This may involve placing a client identifier in the room so that others know of the client's condition, just as a client with swallowing problems has a sign posted above the bed. A client with vision loss may be able to see some things, so the nurse should ask questions to determine how much the client can see, and since the hospital room is a new environment for the client, he will need assistance when getting out of bed. E. Remind the client to call for help whenever getting out of bed--A client with vision loss may need some assistance while in the hospital and the nurse would need to take measures to uphold the client's safety. This may involve placing a client identifier in the room so that others know of the client's condition, just as a client with swallowing problems has a sign posted above the bed. A client with vision loss may be able to see some things, so the nurse should ask questions to determine how much the client can see, and since the hospital room is a new environment for the client, he will need assistance when getting out of bed. D. Place an identifier sign above the client's bed notifying staff of the client's vision loss--A client with vision loss may need some assistance while in the hospital and the nurse would need to take measures to uphold the client's safety. This may involve placing a client identifier in the room so that others know of the client's condition, just as a client with swallowing problems has a sign posted above the bed. A client with vision loss may be able to see some things, so the nurse should ask questions to determine how much the client can see, and since the hospital room is a new environment for the client, he will need assistance when getting out of bed. C. Keep the client's belongings in the same place and do not move them unnecessarily--A client with vision loss may need some assistance while in the hospital and the nurse would need to take measures to uphold the client's safety. This may involve placing a client identifier in the room so that others know of the client's condition, just as a client with swallowing problems has a sign posted above the bed. A client with vision loss may be able to see some things, so the nurse should ask questions to determine how much the client can see, and since the hospital room is a new environment for the client, he will need assistance when getting out of bed.

The clinic nurse is reviewing the plan of care for a 50-year-old client. The nurse recognizes that which of the following is the diagnostic tool of choice for colorectal cancer screening? A. Sigmoidoscopy B. Colonoscopy C. Virtual colonoscopy D. Stool DNA test

B. Colonoscopy This is the preferred screening for colon cancer, because it is a thorough screening that also allows the provider to perform a biopsy on suspicious tissue and remove polyps during the screening. In a client without an increased risk, it is recommended to start routine colonoscopies at age 50, and repeat every 10 years.

Your patient had an ileostomy placed 2 weeks ago. Which bowel pattern would be expected for this patient? A. Continent, controlled output B. Continuous output C. Output once per day D. Output every 2-3 hours

B. Continuous output An ileostomy is placed in the ileum of the small intestine. Therefore output will be liquid or semi-soft and will be continous. As the peristalsis occurs in the small intestine, stool output will be pushed out - therefore clients should expect continuous output throughout the day and night

A nurse is caring for a client with chronic alcohol abuse and is concerned the client may have developed pancreatitis. Which of the following signs are indicative of this? Select all that apply. A. McBurney's Sign B. Cullen's Sign C. Goodell's Sign D. Turner's Sign E. Chadwick's Sign

B. Cullen's Sign Cullen's sign is bruising around the umbilicus that is indicative of pancreatitis or inflammation of the pancreas. D. Turner's Sign Turner's sign is bruising along the flank that is indicative of pancreatitis.

A nurse is caring for a client who is complaining of abdominal pain. Upon inspection of the abdomen, what sign should the nurse be looking for to alert you to potential pancreatitis? A. McBurney's sign B. Cullen's sign C. Battle's sign D. Murphy's sign

B. Cullen's sign Cullen's sign is bruising located in the umbilicus area and indicates potential pancreatitis.

The nurse knows that which of the following most accurately describes the rationale for administering famotidine to a client with gastritis? A. Decrease acid reflux B. Decrease inflammation in the stomach C. Alleviate client pain D. Reduce nausea and vomiting

B. Decrease inflammation in the stomach Famotidine is an H2 receptor blocker that decreases inflammation caused by gastritis by halting the histamine response to the inflamed area.

A nurse is caring for a client with cirrhosis. Which of the following lab values would the nurse NOT expect to see with this disease process? A. Elevated bilirubin B. Decreased PT/INR C. Increased ammonia levels D. Decreased platelets

B. Decreased PT/INR PT and INR would be prolonged (increased), not decreased, because of a decreased synthesis of prothrombin in the liver.

A nurse is assessing the dietary intake of an 80-year-old client who is in the healthcare clinic. Which of the following is related to changes associated with eating that contribute to impaired nutrition? A. Increased risk of diarrhea from changes in nutrient absorption B. Decreased intake of fiber because of difficulties chewing these foods C. Increased intake of fluids due to an increased thirst mechanism D. Decreased blood pressure that makes it difficult to sit for long periods

B. Decreased intake of fiber because of difficulties chewing these foods--An older adult client may have a difficult time with getting enough nutrients and may suffer from vitamin deficiencies and chronic constipation. Adults should consume foods that contain fiber, but these foods are often fibrous, which make them difficult to chew. If a client has a hard time chewing these foods and is not getting enough fiber, the nurse can suggest other sources, such as fiber supplements.

A nurse is caring for a client with a gastric ulcer who begins to have active bleeding. Which of the following is the priority nursing intervention? A. Give NSAIDs for pain B. Give blood if low hemoglobin C. Give fluid and electrolytes D. Instruct on proper eating habits

B. Give blood if low hemoglobin The nurse will monitor the client for signs of hypovolemic shock, dehydration, and sepsis, and be prepared to give blood as ordered. Fluid and electrolytes can be given and the nurse should instruct the client on proper eating habits, but these are not the priority in this situation.

An ostomy nurse is caring for a client who is scheduled for surgery for placement of a colostomy. The nurse explains that it is important to choose the most appropriate site for the stoma. Which of the following regarding stoma location is accurate for the nurse to include in teaching? A. Without proper placement, the stoma will be sunken below the skin level B. If the stoma is not in the right place, you may have trouble reaching it to take care of it C. Placement of the stoma in the right location will affect how you feel about having it D. The stoma is more likely to become infected if it is not exactly placed in the proper location

B. If the stoma is not in the right place, you may have trouble reaching it to take care of it One of the most important aspects to consider for the client who will have a stoma is to determine the correct location of the site. A stoma that is poorly positioned may be difficult for the client to reach, which means the client may have a harder time taking care of it. This could lead to a greater risk of skin complications and infection.

An 89-year old client has been diagnosed with a urinary tract infection. Which best describes a psychological effect of this type of infection? A. Paranoia B. Lethargy C. Depression D. Agitation

B. Lethargy-- A urinary tract infection can cause symptoms of urinary frequency and pelvic pain for many clients. In many older adults, altered mental status and lethargy are the only symptoms of a urinary tract infection. The nurse should look for signs of cognitive changes in older adult clients and assess for possible infection when they occur.

A nurse is talking to an overweight client about weight loss strategies. Which describes an initial recommendation target for weight loss that would be implemented for most overweight or obese clients? A. Loss of 3 inches around the waist in 3 months B. Loss of 10 percent of body weight in 6 months C. Caloric intake between 500 and 1000 calories per day D. Weight loss of 3-4 pounds per week

B. Loss of 10 percent of body weight in 6 months Weight loss instruction is a common element of nursing care. It may often be included as part of primary care, but interventions are often necessary at all levels of healthcare for overweight and obese clients. A common recommendation is for the client to lose 10 percent of body weight within 6 months. The amount of weight to lose and the rate at which to lose it will vary depending on the client's condition.

A nurse is working with a client in the primary care clinic who presents with urinary symptoms. This client experiences urine leakage upon coughing, sneezing and climbing stairs. Based on these symptoms, which of the following types of treatment would most likely be prescribed for management of this client's condition? A. Changes in diet B. Pelvic floor physiotherapy C. Anticholinergic drugs D. Intermittent catheterization

B. Pelvic floor physiotherapy This client is experiencing stress incontinence, which involves the leakage of urine during times of increased pressure in the bladder, such as when the client sneezes or laughs. The condition is most often treated by practicing pelvic floor muscle strengthening exercises, anti-incontinence devices and surgery.

A nurse is assessing a client who is being seen for malnutrition associated with a history of anorexia. Which of the following characteristics of the nervous system would the nurse expect to see in this client? A. Dilated pupils B. Peripheral neuropathy C. Confusion and stupor D. Hyperactive reflexes

B. Peripheral neuropathy Anorexia is the purposeful withholding of food, over for means of control over the body and for weight loss. A client with anorexia may suffer significant malnutrition from decreased intake. Malnutrition can manifest throughout different body systems. Neurological symptoms of anorexia include peripheral neuropathy, hyporeflexia and motor weakness.

The nurse is caring for a female client of childbearing age who presents with infrequent periods, increased acne, facial hair, and obesity. Which of the following conditions do you expect given these assessment findings? A. Pelvic inflammatory disease B. Polycystic ovarian syndrome C. Endometriosis D. Uterine cancer

B. Polycystic ovarian syndrome These assessment findings are consistent with this disease process.

An 86-year-old client has developed protein energy malnutrition as a result of poor eating habits and difficulties with chewing and swallowing. Which best describes how poor nutrition contributes to skin breakdown in an elderly client? A. Poor nutrition is associated with slow and nonhealing wounds B. Poor nutrition affects keratinocytes, which increases the risk of sloughing C. An elderly client secretes less aldosterone, which dries out the skin D. The client has less energy for activities and is less mobile

B. Poor nutrition is associated with slow and nonhealing wounds--Advancing age can increase the risk of nutrient deficiencies and overall malnutrition in older adults. Malnutrition is correlated with a higher risk of developing skin breakdown and non-healing wounds, although the point at which diminished nutrient reserves impact a client's skin integrity is unclear. The nurse should understand the correlation between poor nutrition and skin breakdown and potential wound development in the elderly client.

A client with diverticulosis had a colostomy placed yesterday. Which of the following assessment findings would be the MOST concerning to the nurse? A. Pale-pink stoma B. Purple stoma C. Red stoma D. Red skin around stoma

B. Purple stoma This is the most concerning finding as it indicates severe ischemia and possibly even strangulation of the stoma. This needs to be addressed immediately.

A client has GERD. What changes should the nurse recommend to improve symptoms? Select all that apply. A. Raise the foot of the bed 4-6 inches B. Quit drinking alcohol C. Lose weight D. Eat large meals E. Quit smoking

B. Quit drinking alcohol Alcohol can cause irritation and increase acid reflux. C. Lose weight Excess weight increases abdominal pressure, making stomach acid leakage or backflow more likely to occur. E. Quit smoking For the management of gastroesophageal reflux disease (GERD), the head of the bed should be raised 4-6 inches, the client should eat small meals, quit smoking, quit drinking, lose weight, and should not eat within four hours of bedtime. Smoking can cause irritation and worsen GERD.

An 80-year-old client is confused because he has developed a urinary tract infection. The client cowers and screams when the nurse enters the room. Which action of the nurse best demonstrates that she is attempting to minimize the client's fear in this situation? A. Explain to the client about his condition and provide information in printed form B. Remain calm and continue to try to reorient the client C. Tell the client that everything is okay and he will be just fine D. Help the client to verbalize that he is not afraid of the nurse

B. Remain calm and continue to try to reorient the client A client who is confused may be afraid of once-familiar people or the healthcare providers caring for him. In this case, the nurse probably cannot reach the client by giving him information to read that will help him with his diagnosis. Instead, the nurse should talk to the client calmly and try to reorient them if possible.

The nurse is caring for a client who has chronic kidney disease. Which of the following medications would the nurse question? A. Calcium gluconate B. Spironolactone C. Kayexalate D. Insulin

B. Spironolactone This is a medication that should not be given to a client with kidney disease. Hyperkalemia is a major concern, and since spirolactone is a potassium-sparing diuretic it should NOT be used in clients with CKD.

An older adult client is often incontinent of stool in his bed. He has started to develop an area of skin breakdown on his sacrum. In which ways does stool incontinence contribute to skin breakdown? Select all that apply. A. Stool that is on the skin leads to fissures in the skin tissue B. Stool contains certain enzymes that contribute to skin breakdown C. Skin products applied to the skin do not protect against stool D. Frequent cleaning of the perianal area can increase friction and pressure to the area E. A mix of stool and urine changes the consistency of the excretion to become more caustic to skin

B. Stool contains certain enzymes that contribute to skin breakdown--Stool contains certain enzymes that can contribute to skin breakdown. E. A mix of stool and urine changes the consistency of the excretion to become more caustic to skin--Stool becomes more caustic due to the mixture of chemicals found in urine and stool. D. Frequent cleaning of the perianal area can increase friction and pressure to the area-- A client who is incontinent of stool is at greater risk of skin breakdown than a client who is incontinent of urine only. More frequent cleaning of the perineal area increases friction and pressure, which increases the risk for skin breakdown. Prolonged exposure to moisture causes the skin to become waterlogged and fragile. To improve skin integrity in the client with incontinence, the nurse may apply skin protectants to act as barriers to stool on the skin to prevent exposure to bacteria and chemicals, cleanse skin gently and right away when incontinence occurs, and keep the client's linens dry.

A 92-year-old client with a history of smoking and COPD is found to have developed prostate cancer. The cancer is localized. The nurse knows that prostate cancer in this type of client will most likely be treated in which of the following ways? A. No treatment will be recommended B. Symptom management C. Radical prostatectomy D. Chemotherapy and radiation only

B. Symptom management An older adult who develops prostate cancer is usually treated for symptom management only. This client has COPD, and likely will not tolerate general anesthesia for surgery. Aggressive treatment is rarely sought or recommended in the older adult population.

The nurse is preparing a male client who is about to have a digital rectal exam to check the prostate. The nurse asks the client if the lab has drawn a blood sample yet. What is the reason for the nurse to ask about a blood draw at this time? A. A digital rectal exam can cause vasospasm in the client, making it difficult to obtain a blood sample to test the PSA level B. The digital rectal exam can affect PSA levels, so the blood must be drawn prior to the exam C. The blood sample must be drawn after the digital rectal exam in order to gauge how the prostate responds to the exam D. The prostate exam can cause urinary retention, which decreases the PSA level. This will result in a false-negative PSA level

B. The digital rectal exam can affect PSA levels, so the blood must be drawn prior to the exam A digital rectal exam (DRE) can release prostate-specific antigen into the bloodstream, which temporarily increases the level for 24 hours. If a blood draw occurs after the DRE, the client could have a falsely high level. The blood draw for PSA should happen before the provider's exam in order to provide an accurate baseline PSA level.

A nurse is palpating a client's abdomen to check for an abdominal aortic aneurysm during a physical assessment. Which part of the hand would the nurse most likely use to palpate for this finding? A. The palmar surface B>The pads of the fingers C. The dorsum of the hand D. The ball of the hand

B. The pads of the fingers When assessing a client through palpation, the nurse may use different parts of the hand to find differing signs related to the client's condition. The pads of the fingers are best used for palpating pulsations such as with an abdominal aortic aneurysm, edema, and crepitus, as well as determining moisture content of the skin.

A nurse is caring for a client who is undergoing prostate surgery. Which best describes the elements of informed consent? A> Whether the client understands what the procedure involves B> The risks, benefits, and alternatives to having the procedure C> Whether the client approves of staying in the hospital for the procedure D> The provider's view of whether the client should have the procedure

B. The risks, benefits, and alternatives to having the procedure An informed consent basically describes a consent that the client signs that says the client understands and agrees to having the procedure. The term "informed" means that the client has been educated about the procedure, its risks and benefits, and choices that the client has if the client wants to seek another opinion or have a different procedure.

A patient with malnutrition is receiving TPN for nutrient replacement. The nurse uses aseptic technique when preparing the TPN solution. Which best describes the rationale for this action? A. To ensure the balance of nutrients B. To prevent transmission of pathogens C. To prevent hyperglycemia D. To avoid infusing too much fluid at once

B. To prevent transmission of pathogens TPN often contains high amounts of dextrose, which increases the risk of bacterial infection. If bacteria proliferate in the central venous site of infusion, the patient can develop sepsis. The nurse must use strict aseptic technique when preparing TPN for administration.

The nurse is caring for an infant who has just undergone surgery for Hirschsprung's disease. Which of the following statements would be appropriate to tell the infant's parent? A. Your baby will be constipated for several weeks B. Your baby will have a temporary colostomy C. Your baby will have soft stools from now on D. Your baby will be incontinent of stool as an adult

B. Your baby will have a temporary colostomy Following surgery for Hirschsprung's disease, the infant will have a temporary colostomy. When the child is between 17 and 22 pounds, this will be removed and the bowel reconnected.

A 30-year-old female client has been diagnosed with infertility as a result of polycystic ovary syndrome. The client and her husband have been trying to become pregnant for over a year. The provider prescribes clomiphene citrate. Which of the following best describes how this drug can help to achieve pregnancy? A> Clomiphene opens blocked fallopian tubes that may be closed by scarring B> Clomiphene stimulates ovulation when a woman does not ovulate with PCOS C> Clomiphene enhances the cervical environment to allow sperm to enter the uterus D> Clomiphene increases the lining of the uterus to support the growth of a fertilized egg

B> Clomiphene stimulates ovulation when a woman does not ovulate with PCOS-- Clomiphene citrate (Clomid) is a drug used for the treatment of infertility. It stimulates ovulation in a woman's ovaries, which can increase the chance of becoming pregnant. Clomiphene also increases the chance of multiple eggs being released during ovulation, therefore increasing the chance of multiple births.

Which best describes the order in which the nurse should assess a client's abdomen? A> Auscultation, palpation, percussion, inspection B> Inspection, auscultation, percussion, palpation C> Inspection, palpation, percussion, auscultation D> Percussion, inspection, palpation, auscultation

B> Inspection, auscultation, percussion, palpation When assessing any other part of the body, the nurse would normally perform the methods of assessment in the order of inspection, palpation, percussion, and then auscultation. However, when assessing the abdomen, the order of the techniques is different and should be instead done in the order of inspection, auscultation, percussion, and then palpation. This is because performing palpation or percussion could stimulate the patients gastrointestinal tract and the nurse may hear increased or decreased bowel sounds on later auscultation. The nurse should auscultate to hear bowel sounds first before palpation.

A nurse is caring for a client with continuous bladder irrigation in place. The nurse uses a three-way catheter for the irrigation system. Which best describes the purpose of this type of catheter? A> It allows the nurse to determine if the client is bleeding from the bladder neck B> It allows for irrigant to flow in and out of the bladder C> It prevents the client from experiencing pain from the procedure D>It is divided in its uses for fluid administration and for medication administration

B> It allows for irrigant to flow in and out of the bladder A continuous bladder irrigation system is used to infuse irrigant or medication into the bladder and to clear the bladder of clots, debris, and tissue following surgery. The system uses a three-way catheter: one lumen is used for inflating the balloon of the catheter to keep it in place, one is used to infuse fluid into the bladder, and one is used to drain the fluid from the bladder.

Twelve hours after abdominal surgery, the nurse assesses the client and notes restlessness and complaints of thirst. The nurse notes that the client has cool, pale extremities, tachycardia and has become diaphoretic. The nurse notes that the dressing to the surgical site is soaked with blood. Which of the following is the next action the nurse should take? A. Implement safety precautions B. Provide pressure to the surgical site C. Turn the client on the left side D. Call for a code blue

B> Provide pressure to the surgical site This client is experiencing post-op hemorrhage and is at high risk of shock from excessive blood loss. The nurse must apply pressure to the site of bleeding, whether internal or external. The nurse then contacts the provider, provides oxygen if needed, and administers IV fluids and/or blood as ordered. The client may need to be prepared for surgery based on the provider's orders as well.

The nurse is caring for a client who is newly diagnosed with prostate cancer. The client asks the nurse which treatment will be recommended. Which of the following treatments are targeted therapy usually used for this type of cancer? Select all that apply. A> Immunotherapy B> Radiation C> Hormone therapy D> Chemotherapy E> Surgery

B> Radiation--This is targeted therapy used for localized prostate cancer. E> Surgery--This is targeted cancer treatment that is used for both localized and metastatic disease.

The nurse is assessing a client following a varicocelectomy. Which of the following interventions would be inappropriate? A> Ice application B> Straight catheterization C> Scrotal elevation D> Scrotal support

B> Straight catheterization This is not a part of post-operative care following a varicocelectomy procedure. If the client has trouble passing urine, the provider should be notified, but straight catheterization is not done unless there is a special circumstance and an order.

A nurse is caring for a client with herpes simplex virus. Which of the follow statements by the client regarding herpes simplex virus represents a need for further teaching by the nurse? A. "My antiviral medication will help decrease the symptoms I feel during an outbreak" B. "I will need to have a cesarean section so my infant does not get herpes" C. "Once more sores heal up, I don't need to use a condom anymore" D. "Women are more likely to contract genital herpes"

C. "Once more sores heal up, I don't need to use a condom anymore" This statement is incorrect by the client and requires more teaching. Condom use is encouraged, though it is not 100% effective at preventing HSV transmission. Herpes Simplex Virus, or HSV, is contagious even when a patient is asymptomatic.

The clinic nurse is discussing recommended screenings with a 55-year-old male client. The client mentions that his father had prostate cancer and asks what he can do to minimize the risk of getting this type of cancer. Which response by the nurse is correct? A. "Since you have a family history of the disease, you will need to be screened for the PSA antigen as soon as you experience urinary changes." B. "There is nothing you can do to minimize your risk for prostate cancer, so early detection will be important for you. Plan for routine screenings starting at age 65." C. "The biggest risk factors are age and family history, so there is not much you can do to minimize the chance of getting prostate cancer." D. "Diet has been strongly linked to prostate cancer, so it is important for you to avoid consumption of too much red meat, and increase consumption of vegetables."

C. "The biggest risk factors are age and family history, so there is not much you can do to minimize the chance of getting prostate cancer." The only modifiable risk factor for prostate cancer is diet, but the link is unclear. Routine screenings starting between 50-60 years of age is best, as this will provide early detection and treatment for this type of cancer.

The nurse is caring for a client who will undergo surgery for a right-sided colectomy due to colon cancer. The client asks about what changes to expect after surgery. Which is the accurate response? A. "You'll have a colostomy bag for 3-6 weeks following surgery, so I will help you learn how to care for the site and equipment starting prior to surgery." B. "There is an increased risk for constipation following a right-sided colectomy, so you will need to take laxatives during the recovery period." C. "You will need to make sure you are getting enough nutrients because absorption time is shortened due to the lack of bowel." D. "Short-gut syndrome is one of the expected changes following a colectomy. Your GI tract will be expelling contents more rapidly than normal, so you will be on parenteral nutrition to promote bowel rest."

C. "You will need to make sure you are getting enough nutrients because absorption time is shortened due to the lack of bowel." The right-sided colon, or ascending colon, is a place where last-minute nutrients and fluids are absorbed before the stool is eliminated from the body. The client will have changes in nutrient and water absorption following a right-sided colectomy.

The nurse is caring for a 30-year-old female who has three children and does not wish to conceive again. She was recently diagnosed with endometriosis and has ovarian masses as well as severe dysmenorrhea. When exploring surgical treatment options, which of the following surgeries does the nurse expect is indicated in this situation? A. Salpingooophorectomy B. Exploratory laparotomy C. Abdominal hysterectomy with bilateral salpingooophorectomy D. Abdominal hysterectomy

C. Abdominal hysterectomy with bilateral salpingooophorectomy This surgery is indicated when the client has endometriosis, ovarian masses and is done with childbearing.

The nurse is caring for a client admitted to the floor following orchiectomy surgery. Which of the following interventions is most important for this nurse to employ? A. Applying scrotal support only in bed B. Monitoring client's peripheral pulses C. Administering antibiotics D. Monitoring for urinary retention

C. Administering antibiotics Orchiectomy surgery involves removing one or both testicles. After this type of surgery, prophylactic antibiotics are administered.

The client has a chronic peptic ulcer and wants to know the difference between an acute and chronic peptic ulcer. How does the nurse educate the client? A. An acute ulcer is treated with H2 blockers while a chronic ulcer is treated with proton pump inhibitors B. H. pylori is present with a chronic ulcer but not with an acute ulcer C. An acute ulcer is a superficial erosion, while a chronic ulcer extends through the muscular wall of the stomach D. An acute ulcer lasts only a month and a chronic ulcer lasts greater than one month

C. An acute ulcer is a superficial erosion, while a chronic ulcer extends through the muscular wall of the stomach When the erosion in the lining of the GI tract extends through the mucosal wall and muscle in a portion of the GI tract accessible to gastric secretions, it is called a chronic ulcer. Locations include the stomach, pylorus, duodenum and esophagus. An acute ulcer is in the same locations, but is a superficial erosion through the mucosal wall only.

A 17-year-old client needs to undergo a surgical appendectomy. What is the nurse's role in obtaining informed consent? Select all that apply. A. Ensuring that the parents are competent to sign for the client B. Describing the risks and benefits of the procedure to the client C. Answering questions from the client and the client's parents D. Providing information about a power of attorney for the client E. Talking with the client about the procedure

C. Answering questions from the client and the client's parents--A 17-year-old client would be considered a minor and would need a parent present to sign consent for surgery. The nurse's responsibility with informed consent is to determine if the parents are competent enough to sign and answering questions of the parents and the client. A. Ensuring that the parents are competent to sign for the client--A 17-year-old client would be considered a minor and would need a parent present to sign consent for surgery. The nurse's responsibility with informed consent is to determine if the parents are competent enough to sign and answering questions of the parents and the client.

A client has urinary incontinence diagnosed because of instability of the detrusor muscle. Which of the following describes a form of management of this type of incontinence? Select all that apply. A. Self-catheterization B. Bladder training C. Anticholinergic drugs D. Cranberry juice E. Avoiding caffeine

C. Anticholinergic drugs--Incontinence caused by instability of the detrusor muscle in the bladder leads to incomplete bladder emptying and urine leakage. The condition is typically managed with anticholinergic drugs and lifestyle changes, like avoiding caffeine, which stimulates the bladder. B. Bladder training--The client with a weak detrusor muscle can undergo bladder training to increase the time between bathroom trips. E. Avoiding caffeine--Caffeine stimulates the bladder. When it is avoided, a client may demonstrate increased continence.

A nurse is caring for a client with a nasogastric tube and is preparing to administer an enteral feeding. The nurse wants to check that the tube is in the proper place. Which of the following actions is appropriate for checking placement? A. Listen for bowel sounds and check if they are active, hypoactive, or hyperactive B. Inject 10 mL of air through the tube and listen over the stomach C. Aspirate gastric contents and check the pH D. Ask the client to talk while listening to the abdomen with a stethoscope

C. Aspirate gastric contents and check the pH Before administering fluid or medications into the nasogastric tube of a client, the nurse must check for proper placement. Although the nurse may be able to auscultate bowel sounds when injecting air into the tube, the only accepted manner of checking placement is to aspirate gastric contents and check the pH.

A nurse is working on the GI disorders unit of a hospital and has been assigned three clients. Client A has newly diagnosed celiac disease and requires teaching about a gluten free diet. Client B has abdominal adhesions and is rating his pain at a 6 out of 10. Client C had surgery yesterday, complains of increased abdominal pressure and has a small amount of blood in his mouth. Which client should the nurse see first? A. A, C, B B. B, C, A C. C, B, A D. A, B, C

C. C, B, A With prioritization, the nurse must assess which clients require primary interventions that can threaten airway, breathing, and circulation. This nurse should see client C first because this client shows signs of a GI bleed with increased abdominal fullness and the potential for vomiting blood. This would be followed by the assessment and treatment of pain and finally, teaching and education.

The nurse is teaching a client how to avoid getting E. coli gastroenteritis. Which of the following is appropriate for the nurse to recommend? A Avoid drinking tap water B. Cook eggs thoroughly C. Check cooked meat with a food thermometer D. Microwave milk before drinking

C. Check cooked meat with a food thermometer In order to avoid getting E. coli gastroenteritis, meat must be cooked to 145 to 160 degrees. If E. coli is present in the meat, this temperature kills the harmful bacteria.

A client has been diagnosed with gall stones. The nurse knows that which of the following is the appropriate term for this? A. Lithotripsy B. Nephrolithiasis C. Cholelithiasis D. Cholecystitis

C. Cholelithiasis "Chole-" refers to the gall bladder, "Litho-" refers to stones, and "-iasis" refers to a condition. Therefore "Cholelithiasis" is a condition of stones in the gall bladder, or gall stones.

A nurse checks the contents of a client's indwelling catheter and notes that the urine output has been 10 mL/hr for the last four hours. Which response of the nurse is most appropriate? A. Assess the specific gravity of the client's urine B. Help the client to increase oral fluid intake C. Contact the provider and document the information D. Remove the catheter and irrigate the bladder

C. Contact the provider and document the information Most clients should have an hourly urine output of at least 30 mL per hour. Following surgery or when a client has an indwelling catheter, the nurse should regularly assess urine output to ensure that the client has normal kidney function. If the urine output falls well below the normal range of 30 mL per hour, the nurse should contact the provider for further orders.

The nurse is caring for a preoperative infant who has pyloric stenosis. Which of the following is an appropriate action for the nurse to take? A. Feed large amounts of formula at a time B. Keep the infant lying flat after eating C. Correct fluid and electrolyte abnormalities D. Keep track of the stools

C. Correct fluid and electrolyte abnormalities An infant with pyloric stenosis will have projectile vomiting, which can lead to many problems including aspiration, fluid and electrolyte imbalances, and metabolic alkalosis. In caring for a preoperative infant with pyloric stenosis, the nurse should provide small feedings, correct fluid and electrolyte abnormalities and keep the infant upright after feedings.

An immobile client is at risk of aspiration because of difficulty with chewing and swallowing food. Which of the following nursing interventions is most appropriate? A. Offer liquids to drink before the client tries to eat B. Talk to the client while he eats C. Crush medications and put them in applesauce D. Provide oral care before and after the client eats

C. Crush medications and put them in applesauce This is good nursing management for the client with aspiration risk, but it indirectly prevents aspiration. It is more effective for the nurse to crush medications and give them in a pureed substance like applesauce.

A nurse has just inserted a nasogastric tube for continuous suction. The first hour of suctioning, the nurse notes that the client's output is clear and tinged with blood. Which of the following interventions should the nurse perform? A. Irrigate the tube with 20 mL of normal saline B. Contact the provider for orders to remove the tube C. Document the drainage and continue to monitor D. Remove the tube and insert it as an orogastric tube

C. Document the drainage and continue to monitor During the time following placement of a nasogastric tube, a small amount of blood may be noted in the gastric output. As the tube is placed, it may irritate the lining of the nose and the esophagus, leading to a small amount of bleeding. As long as the bleeding is minor and stops within a short period of time, the nurse should document the drainage and continue to monitor. If the nurse notes a copious amount of blood or a coffee bean color to the drainage, the provider must immediately be notified because this could indicate a gastrointestinal bleed.

The nurse is assigned to a client with an acute kidney injury. The nurse understands that hemodialysis will become necessary when which of the following signs or symptoms are demonstrated? A. Malignant hypertension B. Blood pH 7.5 C. Fluid overload with K 6.4 D. BUN 18 mg/dL

C. Fluid overload with K 6.4 This answer indicates multiple symptoms of kidney failure, which means the client has complications. This, or severe fluid overload with refractory hypertension, severe metabolic acidosis or BUN >70 mg/dL indicates the need for hemodialysis.

A 21-year-old client with anorexia has been given the nursing diagnosis of Altered Nutrition: Less than Body Requirements related to an unwillingness to eat and demonstrated by severe weight loss to less than 100 lbs. Which nursing intervention is most appropriate that should be included as part of the client's ongoing assessment? A. Consult a dietitian for nutrition recommendations B. Offer liquid nutrition supplements for added calories C. Have the client document all food intake D. Encourage the client to participate in mild exercise

C. Have the client document all food intake Anorexia is a psychological condition that occurs when a person chooses not to eat and tries to lose weight. The client may lose excessive amounts of weight to a dangerous and unhealthy level. Management of the client with anorexia involves ongoing assessment and continued care, as this condition takes time to overcome. The nurse may ask the client to document all food intake to establish patterns of eating, as this helps to prevent relapses and provides a way to monitor progress.

A client who is in renal failure must start to use dialysis. Which of the following best describes a difference between hemodialysis and peritoneal dialysis? A. Peritoneal dialysis must be performed daily, while hemodialysis can be performed once per week B. Hemodialysis requires a port for a catheter, while peritoneal dialysis does not C. Hemodialysis requires a dialysis machine to filter the blood while peritoneal dialysis uses the client's body for the same function D. Peritoneal dialysis can be performed at home but hemodialysis cannot

C. Hemodialysis requires a dialysis machine to filter the blood while peritoneal dialysis uses the client's body for the same function A client who requires dialysis may use either hemodialysis, which removes the blood, cleans it, and returns it to the body, or peritoneal dialysis, in which fluid is instilled into the abdomen and then removed. With peritoneal dialysis, the client's peritoneal membrane acts as a dialyzer instead of the machine used with hemodialysis.

A nurse is caring for a 15-year-old client who is recovering from an appendectomy. The nurse is assessing the client's level of pain. Which question from the nurse would be most appropriate for determining the intensity of pain the client is experiencing? A. Where is your pain? B. When did the pain start? C. How would you rate your pain on a scale of 0-10? D. What does the pain feel like?

C. How would you rate your pain on a scale of 0-10? The intensity of a client's pain describes how much pain the client feels, whether it is a little or a lot. The nurse can best assess intensity of pain by asking to rate the amount of pain the client is experiencing on a scale of 0 to 10, with 0 being no pain and 10 being the worst pain.

A nurse is caring for a client with chlamydia. The nurse knows that which of the following statements by the client with Chlamydia demonstrate an understanding of the plan of care? A. If I use a condom, I don't need to worry about waiting 7 days after treatment to resume sexual activity B. "As long as I do not have symptoms, I do not need to worry about spreading to my partner" C. I will return in 3 months for a follow-up visit. D. My partner should come in for treatment if they start to show signs of infection.

C. I will return in 3 months for a follow-up visit. Chlamydia is often asymptomatic, clients are instructed to return in 3 months to monitor for re-infection.

The nurse receives report on a client who will be coming to the floor following a TURP (transurethral resection of the prostate). The nurse prepares for the client's arrival by gathering which of the following supplies for continuous bladder irrigation? Select all that apply. A. A urinal B. 3-way blood tubing C. IV pole D. Infusion pump E. 3,000 mL bags of 0.9% saline

C. IV pole The client will have continuous bladder irrigation, and will need an IV pole for hanging the bags of normal saline irrigant E. 3.000 mL bags of 0.9% saline This is the most common size and solution of bladder irrigant.

A client with irritable bowel syndrome has asked the nurse if there is anything that can be done to decrease the frequency of diarrhea. Which of the following advice is appropriate for a client to develop regular bowel functioning? Select all that apply. A. Eat the largest meal of the day in the evening B. Drink 8 to 10 glasses of liquids per day C. Increase intake of fiber D. Decrease activity levels and increase rest E. Utilize narcotics to reduce pain

C. Increase intake of fiber The nurse who works with a client who has irritable bowel syndrome may counsel the client to increase dietary fiber, drink plenty of fluids, limit greasy and fatty foods, and increase activity levels in order to best regulate bowel function. If dietary and lifestyle changes do not assist the client in managing IBS, medications may be used. B. Drink 8 to 10 glasses of liquids per day Chronic diarrhea can lead to dehydration and electrolyte imbalances for the affected client. The client should stay hydrated to avoid further irritation of the bowels.

Which best describes sinography as used to evaluate a stab wound to the abdomen? A. Completing a rectal exam to determine if bleeding has occurred in the large intestine B. Creating a surgical incision around the stab site and then performing a CT scan C. Injecting a contrast agent through a catheter into the wound and taking an x-ray D. Instilling a liter of fluid into the abdomen to analyze for internal bleeding

C. Injecting a contrast agent through a catheter into the wound and taking an x-ray A client with a stab wound will most likely have a puncture hole or wound that makes it difficult to determine the extent of damage beneath the surface of the skin. Sinography is a radiographic procedure that involves instilling a contrast agent into the wound cavity or 'sinus', and then taking an x-ray of the area to determine if there is tissue damage or bleeding under the skin surface.

A nurse is performing bladder irrigation on a client. The catheter has been inserted and has drained the urine from the client's bladder. Which of the following steps is performed next? Select all that apply. A. Allow irrigation solution to empty out of the bladder into a sterile receptacle B. Remove the catheter from the bladder after instilling irrigation solution and allow the client to void C. Instill irrigation solution into the catheter and then remove the syringe and allow the solution to flow out of the catheter D. Instill irrigation solution into the bladder and clamp the catheter for 30 minutes E. Push approximately 1000 mL of irrigation solution through a syringe into the catheter in the client's bladder

C. Instill irrigation solution into the catheter and then remove the syringe and allow the solution to flow out of the catheter--Bladder irrigation involves inserting a catheter into the client's bladder and injecting a certain amount of irrigation solution into the bladder. The solution could be sterile water or saline. A. Allow irrigation solution to empty out of the bladder into a sterile receptacle--After the solution has entered the bladder, the nurse then lets the solution drain out of the end of the catheter tip (the catheter is not attached to a collection bag at this time), and into a receptacle. The process of bladder irrigation removes elements that have been in the bladder that need to be removed, such as mucous or blood clots.

A client is preparing to undergo a colonoscopy for evaluation of polyps. The nurse would perform which of the following actions when assisting during a colonoscopy? A. Provide suction to remove mucous or blood from the procedure B. Insert lubricant into the client's rectum to ease the colonoscope C. Monitor the client for pain and excess bleeding D. Palpate the abdomen during the procedure to guide the scope

C. Monitor the client for pain and excess bleeding A nurse may assist a provider with a colonoscopy to assess the client's colon for abnormalities such as polyps, or to take samples of tissue. The client is given IV sedation, such as midazolam or fentanyl so that the client is sedated but conscious. The nurse's main role during the procedure is to monitor for signs of bowel perforation, including excess bleeding or the client experiencing severe pain. Bowel perforation is one of the most serious types of complications associated with this procedure.

A client in the emergency department reports pain in the left upper quadrant (LUQ) of the abdomen. The nurse knows that which of the following is located in the LUQ? A. Appendix B. Gallbladder C. Pancreas D. Sigmoid colon

C. Pancreas The following are located in the LUQ; left lobe of the liver, stomach, spleen, body and tail of the pancreas, spleenic flexure of the colon, and part of the transverse and decending colon.

A client just returned from the OR after a gastrectomy. What position would be most appropriate for the client after this particular surgery? A. Dorsal recumbent B. High Fowler's C. Semi-Fowler's D. Prone

C. Semi-Fowler's This position is optimal for respiratory gas exchange while minimizing pressure on the incision.

A client who had an indwelling catheter has developed a urinary tract infection that has spread to the kidneys. The nurse should include which of the following information as part of her teaching? A. The client will need surgery for removal of infected tissue B. The client will need to confirm the diagnosis with urine testing, kidney x-ray, and a voiding cystourethrogram C. The client can continue to recover with antibiotics at home D. The client should restrict fluids until the infection has cleared

C. The client can continue to recover with antibiotics at home--An indwelling catheter places a client at a higher risk of a urinary tract infection (UTI). A kidney infection can develop if the UTI is left untreated, when the bacteria ascend from the lower urinary tract. While a kidney infection, or pyelonephritis, can be very serious and may require hospitalization if complications develop, most clients can recover at home on oral medication after a loading dose of IV antibiotics.

A client with a gastrointestinal condition is suffering from chronic constipation. The nurse recommends that the client increase fiber intake to 25 or 30 g per day. Which of the following factors must the nurse consider when counseling the client to increase fiber intake? A. The client may not see a resolution in constipation for several weeks after increasing fiber intake B. The client should choose fruits that contain plenty of fiber, such as bananas and peaches C. The client should increase fiber intake slowly over the course of several days D. The client should get most dietary fiber from supplements, rather than from food

C. The client should increase fiber intake slowly over the course of several days Increased fiber intake can have positive benefits, including reducing levels of LDL cholesterol and regulating bowel elimination. The nurse can encourage the client to increase her fiber intake to 25 or 30 g per day. However, when doing this, the client should increase the amount slowly over the course of several days, because too rapid of an increase in fiber causes stomach upset and excess gas production.

A nurse is providing discharge teaching to a client who has had a colostomy surgically placed during the hospital stay. What information would the nurse most likely include about the stoma for this clent? Select all that apply. A. The stoma may be slightly swollen just after surgery B. The stoma is usually red in color C. The stoma is typically round or oval in shape D. The stoma is painful when touched E. The stoma is usually flat or inverted

C. The stoma is typically round or oval in shape--A client with a new colostomy may be surprised at the appearance of the stoma and requires teaching about what to expect as well as how the stoma may change. The stoma site may be slightly swollen just after surgery, but this should resolve with time. The client should understand what a normal stoma looks like in order to detect potential complications if there is a change in appearance. A red or pink stoma indicates high vascularization, which is normal. If the stoma becomes pale pink, the client may have low hemoglobin and hematocrit levels. B. The stoma is usually red in color---A client with a new colostomy may be surprised at the appearance of the stoma and requires teaching about what to expect as well as how the stoma may change. The stoma site may be slightly swollen just after surgery, but this should resolve with time. The client should understand what a normal stoma looks like in order to detect potential complications if there is a change in appearance. A red or pink stoma indicates high vascularization, which is normal. If the stoma becomes pale pink, the client may have low hemoglobin and hematocrit levels. A. The stoma may be slightly swollen just after surgery---A client with a new colostomy may be surprised at the appearance of the stoma and requires teaching about what to expect as well as how the stoma may change. The stoma site may be slightly swollen just after surgery, but this should resolve with time. The client should understand what a normal stoma looks like in order to detect potential complications if there is a change in appearance. A red or pink stoma indicates high vascularization, which is normal. If the stoma becomes pale pink, the client may have low hemoglobin and hematocrit levels.

A nurse is working with a client who had bowel surgery with placement of a colostomy one month ago. The client has not had any complications following colostomy surgery so far. Which nutritional recommendation should the nurse give that is appropriate for a client in this stage of recovery? A. Try new foods frequently to further improve output B. Measure waist circumference once per week C. Thoroughly chew food D. Limit the amount of fluids consumed

C. Thoroughly chew food A client who has had a colostomy placed and who is not having complications can eat a normal diet and enjoy food as before the surgery. Although this client is still in a relatively early stage of recovery, the nurse should reinforce with the client to chew food thoroughly while eating and drink plenty of fluids, because blockages and dehydration are a common problem related to colostomies.

A client with chronic diarrhea tells the nurse that she wants to start taking probiotics to regulate her digestive system. In which products are probiotics most commonly found? Select all that apply. A. Wheat bread B. Spinach C. Yogurt D. Sauerkraut E. Milk chocolate

C. Yogurt Probiotics are living organisms that benefit a person's gastrointestinal tract. They have been known to help in certain cases of diarrhea. Probiotics are typically found in fermented products, and items such as sauerkraut, gouda, cheddar and mozzarella cheese, miso soup, sourdough bread, dark chocolate, and yogurt. D. Sauerkraut Probiotics are found in fermented products such as sauerkraut.

A client with a history of ulcerative colitis is having surgery for placement of an ileoanal reservoir. The nurse is educating the client about what to expect from the procedure. Which of the following statements by the nurse is correct? A.Most people who have this procedure struggle with chronic constipation B. Once the surgery is done, you will be able to use the pouch right away C.You will most likely need more than one surgery before this process is complete D. After the procedure is complete, your bowel regimen will be very different from a normal bowel regimen

C. You will most likely need more than one surgery before this process is complete A client who needs surgery to place a colostomy may instead have an ileoanal reservoir placed, which uses a section of the intestine to create an internal pouch rather than using an external bag. The nurse should counsel the client about lifestyle changes that will be needed and should explain that placement typically requires at least two surgeries.

After reviewing a client's colonoscopy images, the nurse is preparing discharge teaching. What instructions regarding dietary requirements would be most appropriate for this client? A. Avoid dairy products B. Avoid drinking too much water C. Avoid high fiber foods D. Peppermint can ease an upset stomach

C> Avoid high fiber foods Clients with diverticulosis have small outpouchings within their colon. These patients should eat a low fiber diet, including canned or cooked fruits and vegetables, dairy products, and potatoes.

Which of the following non-surgical treatment options is not indicated for treatment of endometriosis? A> Biofeedback therapy B> Oral contraceptives C> Ice pack application D> Injectable progestins

C> Ice pack application-This is not indicated for treatment of endometriosis. Heat pack application is indicated because it helps with temporary pain relief.

Which of the following terms indicates and infection in the renal pelvis? A. Cystitis B. Glomerulonephritis C. Pyelonephritis D. Folliculitis

. C. Pyelonephritis The term "pyelo" refers to the pelvis, "nephro" refers to the kidneys, and "itis" refers to infection or inflammation.

A nurse is providing dietary discharge teaching to a client who has been diagnosed with cirrhosis who has had an elevated ammonia level. Which of the following statements by the client demonstrate that more education is required? A. "I'll make sure to eat a high-protein diet." B. "I need to make sure I eat a high-calorie diet." C. "I really need to watch my salt intake." D. "I'll go to the grocery store and stock up on nutritional shakes."

A. "I'll make sure to eat a high-protein diet." Ammonia is a byproduct of protein metabolism. Clients with cirrhosis and an elevated ammonia level should be on a low to normal protein diet. This client would require further education.

A nurse is caring for a patient who is in end-stage renal disease and requires dialysis. Choose which alternative best describes the type of diet the nurse should recommend for this patient. A. A diet high in protein B. A diet high in salt C. A diet low in protein D. A diet low in carbohydrates

A. A diet high in protein Many patients with kidney disease require low-protein diets because of the effects of protein on the kidneys. However, when a patient is in end-stage renal disease and requires dialysis, a low-protein diet may not be necessary. In fact, the patient may more likely require a high-protein diet at this stage because of the loss of nutrients from dialysis.

The nurse is caring for a client with an abdominal injury. Which of the following is a typical finding with an injury to this area? A. Abdominal rigidity B. Dyspnea on exertion C. Diarrhea D. Chest pain

A. Abdominal rigidity A typical finding in a client with an abdominal injury is abdominal rigidity, because the client is tensing to avoid pain and pressure to the area. There can also be nausea and vomiting and abdominal distention. Additionally, the nurse may finde resonance over the spleen or liver indicating injury to these organs.

A 61-year-old client has chronic constipation as a result of prescription medication. Which food should the nurse advise the client to increase to help resolve the constipation? A. Acorn squash B. Potato chips C. Ice cream D. Ground beef

A. Acorn squash Constipation is a common condition that results in infrequent bowel movements and passage of hard, dry stools. A client suffering from chronic constipation, whether or not it is related to medication, should increase fluid intake and consume high fiber foods. Examples of high fiber foods include fruits, vegetables, beans, whole grains and seeds.

The nurse is caring for a client in renal failure who has recently learned that she must begin dialysis. Which of the following is an indication that the client would benefit from peritoneal dialysis instead of hemodialysis? Select all that apply. A. An active lifestyle B. Congestive heart failure C. Vascular access failure D. A fear of needles E. Intolerance to hemodialysis

A. An active lifestyle A person with an active lifestyle is a candidate for peritoneal dialysis because there is more flexibility and control for the client to work the dialysis into his or her schedule versus hemodialysis. B. Congestive heart failure Because there is less fluid exchanged in peritoneal dialysis compared to hemodialysis, the client in heart failure benefits from peritoneal dialy C. Vascular access failure When a client has a failing hemodialysis graft, they become candidates for peritoneal dialysis. sis. D. A fear of needles A client who is afraid of needles benefits from peritoneal dialysis, because a catheter is utilized, so the client does not need to have a needle inserted with each dialysis exchange. E. Intolerance to hemodialysis A client who simply cannot tolerate hemodialysis will benefit from peritoneal dialysis, because it is a more gentle form of waste removal, functioning more similarly to the kidneys.

A nurse is giving education to middle schoolers on preventing human papilloma virus (HPV) and knows that which of the following statements are true? Select all that apply. A. An individual might be contagious even if they have not had warts for a few weeks B. Avoiding vaginal sex when a partner has warts helps to prevent the spread of HPV C. Condoms can help to prevent the spread of HPV, but they are not 100% effective D. An individual with HPV might not have symptoms for 8-12 months after they are infected E. Getting the gardasil vaccine helps prevent pre-cancerous growths

A. An individual might be contagious even if they have not had warts for a few weeks HPV is contagious, even once the individual has no visible warts present. E. Getting the gardasil vaccine helps prevent pre-cancerous growths The gardasil vaccine is effective in preventing pre-cancerous growths and warts. C. Condoms can help to prevent the spread of HPV, but they are not 100% effective Condoms help to prevent the spread of HPV between mucous membranes, but they do not protect from warts or lesions in the groin and inner thigh regions.

A client with a paralytic ileus has an order to insert an NG tube and set to low-intermittent suction. Which of the following steps would be included as part of inserting an NG tube? Select all that apply. A. Check the client's coagulation studies prior to insertion B. Test the gag reflex by asking the client to swallow two cups of water C. Insert air into the NG tube with a syringe and auscultate with a stethoscope to verify placement D. Measure the tube from the tip of the nose to the xiphoid process E. Inspect the client's nares for polyps

A. Check the client's coagulation studies prior to insertion The nurse should check coagulation studies, because bleeding tendencies could lead to excess bleeding with insertion of a nasogastric tube. E. Inspect the client's nares for polyps The nurse should assess the situation prior to tube insertion to keep the process as uncomplicated as possible, which involves assessing for signs of obstruction that would make insertion difficult, such as inspecting for polyps in the nares, and determining if one nare is larger than the other for insertion.

The nurse is caring for a client with poorly controlled GERD. The nurse is providing education regarding foods that can exacerbate the condition. Which of the following would be an appropriate food for this client to eliminate? A. Chocolate B. Purine-containing foods like organ meats (liver, kidneys) C. Citrus fruits D. Gluten-containing foods

A. Chocolate Chocolate contributes to GERD symptoms, because it decreases the tone of the esophageal sphincter which worsens the reflux. Other foods that have this effect include coffee, soda, tea, peppermint, and fried or fatty foods.

An 18-year-old has chronic kidney insufficiency and must take an injection of growth hormone. He takes his own injections and his parents bring him to the healthcare clinic every four weeks for follow-up evaluation. What laboratory levels would need to be monitored in a client taking growth hormones? A. Cholesterol levels B. White blood count C. Hemoglobin levels D. Blood gas analysis

A. Cholesterol levels Human growth hormone is often given to pediatric clients with chronic kidney disease, because CKD can cause less than normal growth in children. While growth hormone often works to correct growth abnormalities, it has been shown to affect cholesterol levels. A client who takes regular growth hormone injections will need to have routine cholesterol level testing.

A nurse is performing a gastric lavage on an adult client. The nurse has placed the nasogastric tube, positioned the client, and instilled 200 mL of warm water into the tubing. Which step does the nurse perform next? A. Clamp the instillation tubing and aspirate gastric contents B. Clamp the nasogastric tube and allow the client to rest for 30 minutes C. Turn the client to the opposite side D. Slowly infuse 200 mL of normal saline

A. Clamp the instillation tubing and aspirate gastric contents During a gastric lavage, the nurse instills fluid into a tube that is fed into the client's stomach. After instilling 200 to 300 mL of fluid into the stomach, the nurse should then clamp the inflow tube to prevent more fluid from entering and unclamp the outflow tube to either allow the fluid to flow out, or aspirate the stomach contents with a syringe.

A client is hooked up to low intermittent suction through a nasogastric tube, and the nurse is preparing to irrigate the tube. After assessing the client's abdomen and determining that the tip of the tube is located in the stomach, the nurse draws up 30 mL of normal saline into a syringe. Which best describes the next step in this process? A. Clamp the suction and disconnect the tube B. Attach the syringe to the NG pigtail and administer 30 mL of normal saline C. Assess for kinks or other signs of occlusion in the tubing D. Ask the client to open her mouth and swallow

A. Clamp the suction and disconnect the tube When the nurse is preparing to irrigate a client's NG tube, proper location of the tube tip must first be assessed. Following this, the nurse turns off the suction and disconnects the tube from the suction to instill fluid into the tube. Once fluid is instilled into the tube, the nurse then aspirates the fluid and can reconnect the tubing to suction.

Which of the following lab test results are consistent with menopause? A. Decreased estrogen B. Increased inhibin C. Increased WBC D. Decreased FSH

A. Decreased estrogen This is one of the changes in the body that is consistent with menopause.

The nurse is caring for a client who has been diagnosed with chronic kidney failure. Which of the following aspects of this client's history could have contributed to this? A. Diabetes mellitus B. Hypertension C. Rheumatoid arthritis D. Transient ischemic attack (TIA) E. Generalized anxiety disorder

A. Diabetes mellitus--Diabetes can cause chronic kidney failure, because the high glucose level causes damage to the nephrons, which decreases their ability to filter the blood. B. Hypertension--Hypertension causes too much pressure on the structures of the kidneys over time, causing them to fail. C. Rheumatoid arthritis--Autoimmune disorders can cause chronic kidney failure when the body's immune system begins to attack these organs.

The dialysis nurse is educating a client who will begin hemodialysis. Which of the following statements by the client is correct? A. Dialysate is the fluid in the filter that causes waste to be pulled from my blood B. When I start to experience heart palpitations, I will schedule my next dialysis appointment right away C. Since my dialysis appointments last all day, I will need to plan for working extra on my non-dialysis days D. Effluent is the interstitial fluid that builds up when I need dialysis

A. Dialysate is the fluid in the filter that causes waste to be pulled from my blood Dialysate is an isotonic solution that has specific solutes to cause a concentration gradient between the client's blood and the solution. This causes the potassium and urea, along with other waste products to be filtered out of the blood and discarded.

When assessing a client's level of nutrition risk, the nurse uses the DETERMINE scale to determine if the client is at high risk of malnutrition or poor intake. Which best describes how the nurse would assess the D (disease) portion of the assessment? A. Find out if the client has a diagnosis of a chronic illness B. Ask whether the client has a genetic risk of an inherited disorder C. Assess whether the client has had an illness in the past month D. Ask the client if they are running a fever

A. Find out if the client has a diagnosis of a chronic illness Illness is one factor that can affect a client's nutritional status. Although acute illness can affect intake, chronic illness is much more likely to be associated with poor nutritional intake. The nurse should first determine if a chronic illness is present and then further assess how the illness affects the client nutrient intake.

The nurse is examining a client who reports they have abdominal pain in the right upper quadrant (RUQ). The nurse knows that which of the following is located in the RUQ? A. Gallbladder B. Tail of the pancreas C. Sigmoid colon D. Appendix

A. Gallbladder The following organs are located in the RUQ; most of the liver, gallbladder, duodenum, head of the pancreas, hepatic flexure of the colon, and part of the ascending and transverse colon.

A nurse is planning medication administration for a client who has all of the following oral medications due at 0900: Calcium carbonate, Codeine, Levetiracetam, Metoclopramide. What is the most appropriate action by the nurse? A. Give Calcium carbonate 1 hour after the others B. Give Codeine 30 minutes before the others C. Give Levetiracetam 30 minutes after the others D. Give Metoclopramide 1 hour before the others

A. Give Calcium carbonate 1 hour after the others Calcium carbonate is an antacid, which should always be given 1 hour after other oral medications, otherwise it may impair absorption of those medications. In this case, it would be appropriate to give the other medications at 0830 and the calcium carbonate at 0930.

What information should the nurse gather about a client's diet during the nutrition screening? Select all that apply. A. Have you had any unplanned weight loss or gain in the past six months? B. Do you eat regular meals each day? C. Which are your favorite foods? D. Do you think you should lose weight? E. Do you have any chronic diseases?

A. Have you had any unplanned weight loss or gain in the past six months?--Part of the general admission assessment for nurses includes a nutrition screening to identify which client are at risk for malnutrition. Discovering information about unplanned weight loss, whether the client receives regular meals, and whether the client has any chronic diseases that could contribute to malnutrition helps the nurse determine whether the client is at risk for malnutrition. B. Do you eat regular meals each day?--Part of the general admission assessment for nurses includes a nutrition screening to identify which client are at risk for malnutrition. Discovering information about unplanned weight loss, whether the client receives regular meals, and whether the client has any chronic diseases that could contribute to malnutrition helps the nurse determine whether the client is at risk for malnutrition. E. Do you have any chronic diseases?--Part of the general admission assessment for nurses includes a nutrition screening to identify which client are at risk for malnutrition. Discovering information about unplanned weight loss, whether the client receives regular meals, and whether the client has any chronic diseases that could contribute to malnutrition helps the nurse determine whether the client is at risk for malnutrition.

A client who has been diagnosed with chronic kidney disease is talking to the nurse about his overall health. Which best describes the difference between health promotion and disease management in this situation? A. Health promotion involves the client being active in the care of his health despite his condition while disease management involves seeking treatments and therapies to control the disease B. Health promotion involves the client educating others about his condition while disease management involves the client focusing on his own care needs C. Health promotion means preventing the disease in the first place while disease management involves dealing with the situation once it has occurred D. Health promotion is promoting wellness activities during times of health while disease management involves working with those struggling with a diagnosis

A. Health promotion involves the client being active in the care of his health despite his condition while disease management involves seeking treatments and therapies to control the disease Health promotion and disease management are closely related in the client with a chronic illness. A client with a chronic disease can still participate in health promotion, which involves being active in care of his health despite his disease. Alternatively, disease management seeks to control the disease through therapy and treatment.

Prior to the client's colonoscopy, nursing interventions include which of the following? Select all that apply. A. Instruct the client to avoid consuming red, orange, and purple liquids B. Clear liquid diet until eight hours prior to the procedure C. Give a gallon of GoLYTELY the evening before the procedure D. Assess for allergy to contrast dye E. NPO 12 hours before procedure

A. Instruct the client to avoid consuming red, orange, and purple liquids The client should avoid red, orange and purple liquids because this can be mistaken for blood during the colonoscopy. B. Clear liquid diet until eight hours prior to the procedure The nurse gives GoLYTELY the evening before the procedure and allows for a clear liquid diet up until 8 hours pre-procedure. C. Give a gallon of GoLYTELY the evening before the procedure The nurse gives a gallon of GoLYTELY the evening before the procedure and allows for a clear liquid diet up until 8 hours pre-procedure.

A client needs to undergo a hemicolectomy. The nurse provides information to the client and asks questions during a pre-operative phone call. The nurse asks if the client has any religious preferences to be considered during the client's stay. The client says, "That's personal information. I do not want to share that with you." Which response from the nurse would most likely explain this client's rights in this situation? A. It is our policy to ask so that we could accommodate any preferences, but you do not have to tell me B. I understand it is personal, but I need to know to be able to care for you correctly C. I have to ask this because it is required that you tell me your religious preferences D. I'm sorry if I offended you. I hope you decide to still have your surgery

A. It is our policy to ask so that we could accommodate any preferences, but you do not have to tell me At the time of pre-admission, a nurse may ask a client information that could be considered personal and that the client does not want to answer. The nurse may ask for religious preferences, but the client is not required to provide the information. Giving religious information to the nurse is voluntary so the hospital can provide for some of the spiritual needs of the client.

A recovery room nurse is caring for a client who is recovering from anesthesia after colon surgery. The nurse is providing IV fluids of Normal Saline at a rate of 150 mL/hr to the client. In order to avoid complications associated with fluid overload, which intervention would the nurse most likely perform? A. Maintain IV administration with a fluid pump instead of gravity B. Apply a pressure support sleeve to the IV bag C. Increase the rate of the IV for the first hour and then turn it down to a very low rate D. Elevate the extremity that has the IV

A. Maintain IV administration with a fluid pump instead of gravity A client recovering from surgery can be at risk of fluid overload if the IV fluids run too fast or are not well controlled. Occasionally clients are given IV fluids by gravity, which must be watched closely to avoid giving too much fluid at once. The nurse can reduce the risk of too high volume administration by using a fluid pump instead of administering fluid by gravity.

A nurse is caring for a client who is complaining of RUQ pain. Which sign should the nurse look for to indicate potential cholecystitis? A. Murphy's sign B. Cullen's sign C. Grey-Turner sign D. Battle's sign

A. Murphy's sign Murphy's sign is severe pain over the RUQ and is indicative of cholecystitis.

A nurse is caring for an adult client who has put out less than 15 mL per hour of urine for the last 6 hours. The nurse knows that this is which phase of acute kidney injury? A. Oliguric B. Polyuric C. Anuric D. Normouric

A. Oliguric The normal urine output is 30-50 mL/hr of urine for an average sized adult. The term "olig/o" refers to "little" or "less than" and "-uric" refers to urine. Therefore putting out very little urine or lower than normal amounts of urine is considered 'oliguria'. This is also known as the oliguric phase of a kidney injury.

The nurse is caring for a client with an ileostomy. During assessment of the client, the nurse notes that the pouch opening is 1/2 inch larger than the stoma site. Which of the following poses a risk for this client? A. Peristomal skin breakdown B. Stoma pain and burning C. Pouch system detachment D. Leakage and odor from the site

A. Peristomal skin breakdown--The opening of the skin barrier and pouch should be just 1/8 inch larger than the stoma. If the opening is any larger than this, the fecal matter will irritate the surrounding skin and can cause skin breakdown. When applying the skin barrier, the nurse should ensure that the skin around the stoma is clean and dry. The stoma is measured, and an opening 1/8 inch larger is applied to the skin using a skin barrier paste. The bag is then attached to the barrier.

A nurse is caring for a client who is recovering from abdominal surgery. The client has developed a cough and at once tells the nurse, "I just felt something pop." The nurse checks the incision site and notes the appearance of bowel loops through the wound opening. What should the nurse do first? A. Place the client in the low Fowler's position and with knees slightly flexed B. Pour sterile saline solution over the wound and into the abdominal cavity C. Don sterile gloves and attempt to keep the intestinal contents within the cavity D. Cover the abdomen with a warm blanket

A. Place the client in the low Fowler's position and with knees slightly flexed The priority nursing action when a client has wound dehiscence and evisceration is to place the client in a low Fowler's position with knees slightly flexed and tell them to rest quietly while the nurse immediately calls for help. The nurse stays with the client while asking other staff members to contact the provider and get supplies. Once the supplies are available, the nurse should place a saline-soaked gauze over the incision site to protect the organs from exposure.

A nurse is caring for a client who has been diagnosed with cirrhosis. The client has been prescribed corticosteroids. Which information describes how this drug helps the cirrhotic liver? Select all that apply. A. Reduction in inflammation B.Anemia C. Suppression of immune system liver destruction D. Steady glucose levels E. Polyuria

A. Reduction in inflammation Corticosteroids are often prescribed for clients with cirrhosis to suppress inflammation. C. Suppression of immune system liver destruction Corticosteroids are often prescribed for clients with cirrhosis to slow immune-mediated liver destruction.

The community health nurse is teaching a class on early detection of testicular cancer. The nurse accurately teaches the class all of the following points except which? A. Self-exams should begin at age 20 B. The sooner testicular cancer is detected, the less aggressive treatment will be needed C. A feeling of fullness in the pelvis is a sign of later stages of testicular cancer D. A hard, non-movable lump is concerning for cancer

A. Self-exams should begin at age 20 This is not an accurate teaching point. Testicular self-exams are recommended to start at puberty.

The nurse is repositioning a client diagnosed with pelvic inflammatory disease in bed. Which of the following positions helps with drainage of infection? A. Semi-fowler's position B. Sim's position C. Supine position D. Trendelenburg position

A. Semi-fowler's position This helps with drainage of infection.

A nurse is caring for a client who is suffering from a gastric ulcer. Which of the following is an appropriate nursing intervention? Select all that apply. A. Take a proton pump inhibitor B. Take a histamine blocker C. Drink caffeinated beverages D. Eat small, frequent meals E. Drink plenty of milk

A. Take a proton pump inhibitor Histamine blockers or proton pump inhibitors should be given to reduce stomach acid. B. Take a histamine blocker Histamine blockers or proton pump inhibitors should be given to reduce stomach acid. D. Eat small, frequent meals Small, frequent meals are less likely to cause irritation to an ulcer than large meals.

The nurse is caring for a client who is recovering from an inguinal hernia repair. Which of the following nursing interventions are appropriate? Select all that apply. A. Teach turning and deep breathing B> Apply heat to the swollen scrotum C. Provide scrotal support D. Avoid a high-fiber diet E. Relieve urinary retention

A. Teach turning and deep breathing The client should have ice to the scrotum, and should be taught turning and deep breathing without coughing, in order to reduce pressure to the area. They should avoid lifting and pulling/pushing for 6-8 weeks. C. Provide scrotal support Scrotal support is necessary to avoid pulling on the weakened area. E.Relieve urinary retention A herniorrhaphy is performed to push contents of the abdomen back through the weakened wall through which it is has protruded. After a herniorrhaphy to the inguinal area, the client may experience urinary retention, which should be relieved with a catheter if necessary.

A nurse is assisting a provider with obtaining informed consent for a client who is preparing to undergo a laparoscopic cholecystectomy. What must the nurse consider when determining if the client is competent to sign the informed consent? A. The client has a low level of health literacy B. The provider does not have much time to give information about the procedure C. The client is feeling stress about the upcoming procedure D. The client does not ask any questions about the procedure

A. The client has a low level of health literacy When a client undergoes a surgical procedure, the provider's role is to explain the procedure, as well as its risks, benefits, and alternatives of the procedure. The nurse's should note whether the client is competent to sign the consent and that the client understands what he is signing. A client with a low level of health literacy may not understand the provider's explanation or the information on the form.

A 57-year-old client is preparing to undergo a total hysterectomy. The surgeon has discussed the procedure with the client, including its risks, benefits, and alternatives, and the client has asked a few questions before signing the informed consent document. Which of the following demonstrates that the nurse's role has been completed in obtaining the informed consent? A. The nurse determines that the client is mentally competent to make decisions B. The nurse signs the form for the client C. The nurse expands on what the provider said and adds pertinent details about further risks and benefits of the surgery D. The nurse verifies that the client truly needs the procedure

A. The nurse determines that the client is mentally competent to make decisions The consent is the form that the client signs that says that she agrees to undergoing a procedure. Consent is considered to be informed consent when the client has been informed about the risks surrounding the procedure and the benefits of having it. The provider describes this information to the client. The nurse's role is to ensure that the client is competent to sign and that she understands the procedure.

A nurse is caring for a client who has developed a surgical site infection following surgery for an open cholecystectomy. Which of the following factors contribute to infection rates in surgical wounds? Select all that apply. A. The time between the client's skin preparation and the actual surgery B. The temperature of the client during surgery C. The client having diabetes or being immunocompromised D. The presence of an existing infection before surgery E. The pain medications used during surgery

A. The time between the client's skin preparation and the actual surgery If there was a longer time between skin prep and actual surgery, this exposes the affected area to bacteria and viruses that cause an infection. Additionally, the hospital staff has a responsibility to maintain asepsis during care and monitor for signs of infection. C. The client having diabetes or being immunocompromised If the client has a condition that predisposes them to infection, such as diabetes or an immunocompromised status. D. The presence of an existing infection before surgery Surgical site infections are a type of healthcare associated infection (HAI) that cause an increased burden on the client and hospital, increased the length of stay and increased morbidity and mortality. A client is at greater risk of developing a surgical site infection if their body or a wound was contaminated prior to surgery,

A client presents to the emergency department complaining of right upper quadrant pain which worsens with palpation. The client has clay-colored stools. Which of the following tests will likely be ordered? Select all that apply. A. Ultrasound B. Complete blood count (CBC) C. Influenza test D. Urinalysis E. CT scan

A. Ultrasound An ultrasound will give important information about the severity of the gallstones. B. Complete blood count (CBC) A CBC will help the provider know whether there is an increase in WBCs or not. D. Urinalysis A UA is part of the workup for gallstones. E. CT scan RUQ pain and clay colored stools are classic signs of gallstones, so the client should be checked for increased bilirubin in both blood and urine, increased WBCs in the blood, and undergo imaging such as CT scan or ultra sound.

A 63-year-old female patient complains of urinary retention and an inability to empty her bladder. The nurse understands that the client is likely experiencing which of the following conditions? Select all that apply. A. Uterine prolapse B. Vaginal pemphigus C. Vulvovaginitis D. Urethral stricture E. Bladder tumor

A. Uterine prolapse Uterine prolapse can lead to urinary tract obstruction. D. Urethral stricture A urethral stricture is one of the common causes of urinary tract obstruction in women. E. Bladder tumor Based on the client's symptoms of retention and an inability to empty the bladder completely, the nurse knows she is experiencing urinary tract obstruction. Urinary tract obstruction develops as a result of something blocking the flow of urine. Some common causes of obstruction in women include uterine prolapse, urethral stricture, a bladder tumor, an ovarian cyst, uterine fibroids, or a cystocele. Treatment of urinary tract obstruction centers on correcting the cause of the blockage, and often involves surgical correction of the underlying cause.

A provider has ordered a serum creatinine test for a client who is being assessed for chronic kidney disease. The client asks the nurse about the test. Which of the following responses correctly explains serum creatinine? A. We are testing to see if your kidneys are able to excrete waste through your urine B. The provider ordered this test to check for an infection C. This test tells us if you are releasing excess sugar in your urine D. This will tell us if your kidneys are damaged and you will need dialysis

A. We are testing to see if your kidneys are able to excrete waste through your urine A creatinine test is a test of kidney function that measures the amount of creatinine in the bloodstream. Creatinine is a waste product of creatinine, which is produced by the muscles for energy. Creatinine is removed from the body by the kidneys, and increased levels indicate that the kidneys are not able to excrete normal amounts of creatinine from the body. An increased serum creatinine is not seen until there is at least a 50% renal function loss.

A nurse is counseling a client who has abdominal pain. The client says to the nurse, "My father died of colon cancer and now that I'm over 50, I'm sure that I have it, too!" Which of the following responses from the nurse is most appropriate? A. Your age and family history do put you at greater risk, but we should look at your test results before coming to further conclusions B. You are experiencing symptoms consistent with colon cancer. Because of your family history, we should run some more tests C. Having a family member with colon cancer will not increase your risk of developing it D. Just because your father had it does not mean that you will get it, too. Everyone is different

A. Your age and family history do put you at greater risk, but we should look at your test results before coming to further conclusions The client in this situation understands the higher risk of colorectal cancer because of familial history of the condition, as well as the fact that being over 50 years old increases risk. However, it does not mean that this is the cause of the abdominal pain. The nurse should talk with the client about risks but also focus on the current situation.

The nurse is providing discharge teaching to a client who has been treated for a testicular torsion. Which of the following is an incorrect teaching point? A. Heat application for pain control B. Post-operative care C. Instruction on use of scrotal support D. Importance of prompt intervention

A.Heat application for pain control Ice and pain medication should be used for comfort following testicular torsion surgery, not heat.

A 19-year-old client presents to the emergency room with pain in the right lower quadrant. The nurse knows that which of the following is located in this area? A> Appendix B> Gallbladder C> Sigmoid colon D> Pancreas

A> APpendix

While completing an intake assessment for a 25-year-old client for a routine exam, the client tells the nurse about being treated for a chlamydia infection 3 months prior. The nurse anticipates which of the following interventions? A. Chlamydia and gonorrhea screening B. PAP smear C. Pelvic exam D. Pregnancy test E. One-time dose azithromycin PO

A> Chlamydia and gonorrhea screening The nurse can anticipate a pelvic exam, as the client recently had an STI infection. Chlamydia and gonorrhea screening is correct because patients aged 25 and under are screening annually for chlamydia and these STIs usually coexist. Routine care for a patient treated for Chlamydia includes a 3-month follow-up appointment to check for re-infection. C> Pelvic exam--The nurse can anticipate a pelvic exam, as the client recently had an STI infection. Chlamydia and gonorrhea screening is correct because patients aged 25 and under are screening annually for chlamydia and these STIs usually coexist. Routine care for a patient treated for Chlamydia includes a 3-month follow-up appointment to check for re-infection.

A client receiving medication as an anti-infective starts showing signs of nephrotoxicity. The nurse knows that which of the following are signs of nephrotoxicity? A> Decreased urine output B> Low blood pressure C> Hearing loss D> Fluid retention E> Increased urine output

A> Decreased urine output Damaged renal cells cause a decrease in urine production. D> Fluid retention Damaged renal cells cause fluid retention.

A client has been ordered to receive a voiding cystourethrogram because he has difficulty emptying his bladder. Which of the following actions would take place during this test? Select all that apply. A. The client will lie on a table during the test B> Images will be taken while emptying the bladder C> Contrast dye is administered into the bladder for the test D> The nurse inserts a catheter into the client's bladder E> The client will have to turn from side to side during the test

A> The client will lie on a table during the test--A voiding cystourethrogram is a radiological test to visualize the bladder and urethra while the client empties the bladder. The nurse inserts a urinary catheter and the contrast dye for the test is instilled through the catheter to best visualize the structure of the bladder and post-void residual. D> The nurse inserts a catheter into the client's bladder--A voiding cystourethrogram is a radiological test to visualize the bladder and urethra while the client empties the bladder. The nurse inserts a urinary catheter and the contrast dye for the test is instilled through the catheter to best visualize the structure of the bladder and post-void residual. C> Contrast dye is administered into the bladder for the test--A voiding cystourethrogram is a radiological test to visualize the bladder and urethra while the client empties the bladder. The nurse inserts a urinary catheter and the contrast dye for the test is instilled through the catheter to best visualize the structure of the bladder and post-void residual. B> Images will be taken while emptying the bladder--A voiding cystourethrogram is a radiological test to visualize the bladder and urethra while the client empties the bladder. The nurse inserts a urinary catheter and the contrast dye for the test is instilled through the catheter to best visualize the structure of the bladder and post-void residual.

The nurse is discussing colon cancer risks with a 40-year-old client. Which of the following are modifiable factors that contribute to an increased risk for colon cancer? Select all that apply. A> Inflammatory bowel disease B> Advanced age C> Smoking D>An inactive lifestyle E> A diet high in alcohol consumption

An inactive lifestyle Persons who lead an active lifestyle are less likely to get colon cancer than those who lead a sedentary lifestyle. A diet high in alcohol consumption Diet plays an important role in cancer prevention. A client whose diet includes a high consumption of alcohol and red meat has an increased risk for developing colon cancer. Smoking Smoking increases the risk of developing many types of cancer, including colon cancer.

A client presents with complaints of nausea, vomiting, and abdominal pain for the last 12 hours. The client states to the nurse, "I have had gastritis before and think I might have it again." Which of the following statements from the client would lead the nurse to believe it is something other than gastritis? A. "I drink a 6 pack of beer every night" B. "I hurt my knee at work last week and have been taking 400mg of Ibuprofen every 6 hours ever since" C. "My healthcare provider told me I have had H. pylori before" D. "The pain is a 7/10 in the lower right area of my abdomen."

D. "The pain is a 7/10 in the lower right area of my abdomen." This is correct. RLQ pain does not suggest gastritis. Pain associated with gastritis is most often in the mid-upper and LUQ of the abdomen. RLQ pain is more likely associated with acute appendicitis.

A client is scheduled for a proctocolectomy. The nurse is explaining the procedure to the client. Which of the following is the best explanation by the nurse? A. "A tube will be inserted through a hole in your colon" B. "You will receive a stoma in your small intestine" C. "A camera will be inserted into your colon, via your rectum" D. "Your colon and rectum will be removed surgically"

D. "Your colon and rectum will be removed surgically" The suffix "-ectomy" refers to surgical removal of a structure, while "procto-" refers to rectum and "colo-" refers to the colon. A proctocolectomy refers to the colon and rectum being removed surgically.

The nurse is caring for a client with a hiatal hernia. Which of the following is an appropriate treatment? Select all that apply. A. Recommend a high fat diet B. Teach the client to avoid straining C. Teach the client to avoid anticholinergics D. Administer antacids E. Recommend eating large meals

D. Administer antacids--A hiatal hernia occurs when the stomach protrudes through the diaphragm. The client will experience heartburn, and therefor antacids are appropriate to administer. B. Teach the client to avoid straining--The client should be taught to avoid straining, as this can worsen the hernia. C. Teach the client to avoid anticholinergics--The nurse should educate the client to avoid anticholinergics, as these slow gastric motility.

A client is being dismissed from the hospital after recovering from abdominal surgery. The client is an older adult who lives alone and has few measures of support. The nurse contacts a social worker for support and resources. Which of the following describes how the social worker advocates for a client being discharged from the hospital? Select all that apply. A> Acting as a spiritual advisor to provide holistic health B> Communicating with a senior day program for the client to attend C> Contacting a pain clinic to follow up with the client if pain is an issue D> Arranging for Meals on Wheels for the client E> Assisting the client to complete activities of daily living

D. Arranging for Meals on Wheels for the client A social worker is a member of the interdisciplinary team who is a valuable resource for finding services and resources for the client. In this case, the social worker could be available to make arrangements for community services for the client to improve his quality of life. C. Contacting a pain clinic to follow up with the client if pain is an issue A social worker is a member of the interdisciplinary team who is a valuable resource for finding services and resources for the client. In this case, the social worker could be available to make arrangements for community services for the client to improve his quality of life.

The dialysis nurse has initiated a hemodialysis session with the client. Which of the following is most important to monitor? A. Nausea B. Fever C. Level of consciousness D. Blood pressure

D. Blood pressure During hemodialysis, a client has large fluid volume shifts. The client begins dialysis with fluid overload, and the dialysis will pull of a large volume of fluid. These clients can experience hypotension so it is important for the nurse to monitor blood pressure.

Before starting a session of hemodialysis, a nurse assesses her client's AV fistula. The nurse listens to the fistula with a stethoscope and notes a swishing sound under the skin. Which action of the nurse is most appropriate? A. Raise the client's arm above the level of his heart B. Cleanse the AV fistula site and auscultate again C. Contact the provider to order an ultrasound of the site D. Check for distal pulses and prepare to start the dialysis procedure

D. Check for distal pulses and prepare to start the dialysis procedure An arteriovenous (AV) fistula is used for hemodialysis as a form of vascular access. The nurse should work carefully with the AV fistula site, as this is the client's access for hemodialysis. Prior to the next treatment, the nurse may listen to the site with a stethoscope to determine if a bruit is present, which manifests as a swishing sound under the skin. This is a normal finding, so in this case, the nurse would continue with the rest of the assessment.

A nurse is performing an intake assessment on an older adult. While taking the client's history, the nurse discovers that the client used over-the-counter laxatives for years in order to promote bowel regularity. Based on the nurse's knowledge of laxative use, the nurse knows that in this situation, the client is at risk for which of the following? A. Diarrhea B. Colon polyps C. Malabsorption of vitamins D. Chronic constipation

D. Chronic constipation Some adults use laxatives on a regular basis in order to promote bowel regularity and to reduce instances of constipation. Unfortunately, chronic laxative use can have the opposite effect and can therefore increase the risk of constipation in a client. The nurse should counsel the client that laxatives are meant to be used only on a short-term basis and never for the long term.

A client who has been utilizing peritoneal dialysis for 1 month presents to the clinic with complaints of abdominal pain and nausea. The nurse notes a distended abdomen. Which of the following interventions will the nurse need to do first? A. Instill dialysate for peritoneal dialysis B. Intraperitoneal administration of ampicillin C. Admit the client for 24 hour observation for worsening signs and symptoms D. Collect PD fluid for analysis

D. Collect PD fluid for analysis This client is demonstrating signs of peritonitis. The nurse would expect to swab the collection site and collect PD fluid for analysis first. After the cultures have been obtained, the nurse will expect to start antibiotics.

During a head-to-toe assessment, the nurse knows which of the following findings is suggestive of a gonorrhea infection? A. Wart-like lesions on client's inner thigh B. Open, "kissing" lesions on client's penis and scrotum C. Diffuse rash across the client's back D. Creamy, white exudate from client's penis

D. Creamy, white exudate from client's penis Creamy, white exudate is correct because this type of discharge is a trademark symptom of gonorrhea infection.

A nurse is caring for a client who recently had an NG tube placed. Before starting the first enteral feeding, the nurse wants to check for proper placement. Which of the following techniques is most appropriate to confirm placement? A. Inject 10 mL of normal saline into the tube and listen over the chest B. Inject 10 mL of air into the tube while auscultating over the abdomen C. Ask the client to open his mouth and check the back of the throat D. Draw back a small amount of gastric fluid and test the pH

D. Draw back a small amount of gastric fluid and test the pH--Aspirating a small amount of gastric contents and checking the pH is the best answer out of the options listed, although ideally an X ray should be taken to confirm placement. When a client has an orogastric or nasogastric tube in place, it is essential that the nurse check for proper placement of the tube to ensure that the tip is in the stomach and not the lungs. The nurse absolutely must verify placement before administering formula or medications through the tube.

Which of the following hormonal abnormalities prevents the release of an egg during a menstrual cycle in polycystic ovarian syndrome? A. Normal estrogen levels B. Low testosterone levels C. Low thyroid levels D. Excess androgen levels

D. Excess androgen levels Too much androgen prevents release of an egg during a menstrual cycle in polycystic ovarian syndrome.

The nurse is working with a client who has peptic ulcer disease. Which of the following labs is important to monitor with this condition? A. Lactic acid B. Magnesium C. Procalcitonin D. H/H

D. H/H In PUD, bleeding is a concern, so monitoring the H/H will alert the clinician of developing or worsening bleeding.

A nurse is performing an intake assessment on a new client. The nurse reaches the nutrition portion of the assessment and wants to learn about the client's food intake. Which of the following questions would be most appropriate to gain information about a client's intake? A . What kinds of foods do you dislike? B. Which medications are you taking that are affected by food? C. Do you consume caffeine? D. How many meals and snacks do you eat in a 24-hour period?

D. How many meals and snacks do you eat in a 24-hour period? The nutrition portion of the assessment is important to determine if the client is healthy or if there are issues with food and nutrient intake that can adversely affect the client's health. To assess nutritional intake, the nurse should ask the client questions about how many meals and snacks are consumed over a 24-hour period. While the other questions may elicit some information, they do not provide a sample of total intake.

The nurse is caring for a client who has been diagnosed with nephrotic syndrome. Which of the following assessment findings would be inconsistent with this diagnosis? A. Edema B. Hypoalbuminemia C. Proteinurea D. Hyperalbuminemia

D. Hyperalbuminemia This question is asking for which assessment finding is NOT expected in the client with nephrotic syndrome. The client with nephrotic syndrome will have a LACK of serum protein. This is due to excessive loss of protein through the urine due to damaged kidneys. Hyperalbuminemia is an excess of protein in the blood, which is inconsistent with nephrotic syndrome.

A client has been hospitalized for symptoms of abdominal pain and diarrhea following exacerbation of stress. The nurse is teaching the client about ways to minimize environmental stress in her life. Which statement from the client indicates that teaching has been effective? A. I will try to eat more fat in my diet since my body is able to digest it easily B. I need to remember to check my voicemail instead of always answering the phone C. I will listen to talk radio in the car to distract myself from my problems D. I am going to organize my home so that I do not have so much clutter lying around

D. I am going to organize my home so that I do not have so much clutter lying around Environmental stressors include noise, air quality, crowding, light levels, clutter. A nurse can educate a client about how best to manage symptoms of stress before the client becomes physically ill. The client can organize her living space and remove clutter, as even seeing excess clutter can sometimes cause a stress response.

The nurse is caring for a client with severe liver cirrhosis. Which of the following nursing interventions is appropriate for this client? Select all that apply. A. Provide a low fat diet B. Supplement with vitamin C C. Restrict fluids D. Monitor coagulation studies E. Provide small frequent feedings

D. Monitor coagulation studies Cirrhosis occurs when liver cells are damaged and create scar tissue in the liver. This leads to portal hypertension, and decreases liver function. In caring for a client with severe liver cirrhosis, some appropriate nursing interventions include managing shunts to relieve portal hypertension, monitoring coagulation studies, supplementing with B complex vitamins, and providing small frequent feedings. A. Provide small frequent feedings-- This is appropriate for the client to absorb the most nutrients. C. Restrict fluids--Ascites is a symptom of cirrhosis. The client with a non-functioning liver will be on a fluid restriction.

The nurse is calling a client scheduled for a colonoscopy in three days to give instructions. Which of the following should the nurse include? A. Take a laxative the evening before the procedure B. Consume red and pink jello C. Drink the prep an hour before the procedure D. No solids the day before procedure

D. No solids the day before procedure The clients need to be NPO prior to the procedure accept for the GI prep.

A nurse is caring for a client who has a nasogastric tube in place and is receiving enteral feedings. The nurse must administer formula to the client that has been prepared by the hospital dietary staff and consists of milk-based, blenderized foods. This type of formula is known as: A. Specialized formula B. Modular formula C. Elemental formula D. Polymeric formula

D. Polymeric formula There are four main types of enteral nutrition formulas, which are elemental, semi-elemental, polymeric, and specialized. The type of formula prescribed is based on the client's nutritional needs and whether there is pre-existing malnutrition. Polymeric formulas are typically created from regular foods that have been mixed in a blender. They contain complete nutrition.

A client is brought to the emergency room with complaints of abdominal pain. The nurse performs a focused abdominal assessment. Which of the following is an element of the inspection portion of the exam? Select all that apply. A. There is visible peristalsis B. The client reports consumption of a high-fiber diet C. The client has tenderness D. The abdomen is rounded E. The stoma site is red

D. The abdomen is rounded The inspection portion of an exam includes objective data, obtained by the nurse, while visually inspecting the abdomen. Noting a rounded abdomen is gained from inspecting the abdomen. A. There is visible peristalsis A focused assessment centers on one aspect of the client, such as one certain body part or with specific elements of the client assessment. Visualizing peristalsis is information gained from abdominal inspection. E. The stoma site is red When performing a focused abdominal assessment, the inspection portion of the exam involves viewing the client's abdomen and looking for abnormalities. Noting a reddened stoma site is information gained from the inspection portion of the exam.

A client with limited mobility and a history of urinary incontinence is being admitted to a long-term care facility. Which of the following would the nurse note as relevant to potential issues for this client from the history and assessment? A. Recent history of lice infestation B. History of ear infections C. Abnormal hair growth on the head D. The presence of an endocrine disorder

D. The presence of an endocrine disorder A client's medical history and physical assessment are essential for giving clues about potential complications that could develop. A client with a history of an endocrine disorder, such as hypothyroidism, likely has skin changes associated with the condition that could contribute to skin breakdown. Since the client is already at risk for skin breakdown from immobility and urinary incontinence, this information would prompt the nurse to identify this client as a high risk for skin breakdown and manage appropriately.

A provider has ordered a nurse to perform bladder irrigation on a patient in the hospital. Which of the following describes a purpose of performing bladder irrigation in a client? Select all that apply. A. To promote bladder healing B. To decrease bleeding C. To relieve constipation D. To clear the bladder of blood clots E. To administer pain medications

D. To clear the bladder of blood clots Bladder irrigation is done to remove blood clots from the bladder. It allows for an increased flow of fluid much faster than urine production. A. To promote bladder healing After certain genitourinary procedures such as transurethral resection of bladder tumors (TURB), bladder trauma, open prostatectomy or transurethral resection of the prostate (TURP), the client must have continuous bladder irrigation to wash out and heal the bladder. B. To decrease bleeding Bladder irrigation is a process that involves inserting fluid into the patient's bladder to flush out the bladder. It may be done to "wash out" the bladder of blood clots or bacteria, to decrease bleeding, and to promote healing after some medical procedures, including transurethral resection of bladder tumors (TURB), bladder trauma, open prostatectomy or transurethral resection of the prostate (TURP).

The nurse is educating a client regarding epididymitis. Which of the following pieces of information is not correct regarding this condition? A. Administration of NSAIDs and antibiotics as ordered B. Applying ice for 20 minutes as needed for pain C. Using scrotal support every time the client ambulates D. Treating sexual partners regardless of epididymitis cause

D. Treating sexual partners regardless of epididymitis cause This is not information the nurse should teach a client, because sexual partners should be treated with antibiotics ONLY if the epididymitis is due to a STD.

The nurse is caring for a postoperative client who just underwent an orchiectomy. The nurse understands that which of the following is a risk factor for this client's condition? A. Cycling B. Horseback riding C. Trauma D. Undescended testicle

D. Undescended testicle An orchiectomy is surgical removal of a testicle, and is performed when a client has a testicular tumor. Some risk factors for testicular cancer include a family history of this type of cancer, an undescended testicle at birth, a history of testicular cancer, and young age.

The nurse is caring for a client who was recently diagnosed with testicular cancer. The client appears anxious and asks the nurse questions about his condition. Which of the following statements by the nurse is correct? A> "Testicular cancer is known to spread to other areas, even when caught early. You will need to prepare for chemotherapy right away." B> "You will want to think carefully about having children, because the risk of birth defects increases after treatment for this cancer." C> "You should consider sperm banking, because once treatment begins for this type of cancer your chances of becoming infertile increase to 95%." D> "Testicular cancer is highly treatable, and has a high survival rate."

D> "Testicular cancer is highly treatable, and has a high survival rate." This type of cancer has a high rate of survival, even with metastases present. If caught early, treatment may only consist of an orchiectomy, and often just one dose of chemotherapy. However, the treatment for this condition may impact fertility, so the client must be educated on sperm banking and risk of infertility in order to make informed decisions prior to beginning cancer treatment.

While a nurse is educating students at a middle school health fair, the nurse is approached by a student who shares there are some bumps that look like the pictures of herpes simplex virus. Which of the following responses by the nurse is most appropriate? A Telling the student that abstinence would have prevented her infection B. Requesting the student call her parents to take her to the doctor C. Asking the client if the sexual partner has ever mentioned having genital herpes D. Asking the student if she would be comfortable speaking in a more private place about her concerns

D> Asking the student if she would be comfortable speaking in a more private place about her concerns Requesting the student to talk to the nurse privately is most appropriate so that the nurse can coordinate medical care for the client and evaluate for sexual abuse.

A nurse is assessing a client's nutritional status. In addition to checking the client's weight, which of the following assessments would the nurse include to check for malnutrition? A> Higher-than-normal body temperature B> Increased respiratory rate C> Dullness with percussion over the abdomen D> Fluid retention

D> Fluid retention--Malnutrition can be manifested a number of ways in a client. One common manifestation is a decrease in body weight, but a client who does not take in enough protein in the diet can develop edema in the distal extremities, or generalized edema (anasarca). Protein in the bloodstream helps maintain osmotic pressure, so when a person has a severe deficiency in protein, the body can develop edema.

The nurse is caring for a client and suspects cholecystitis. Upon notification of the provider, the nurse is asked to discover whether the client has a positive Murphy's sign. Which of the following will verify the presence of a positive Murphy's sign in this client? A> Hold the knee flexed 45 degrees. Place one hand on the medial aspect of the knee and pull laterally. Use the other hand to rotate the knee in, and if a click is heard or pain is experienced, the test is considered positive B> Tenderness at the point over the right side of the abdomen that is 1/3 of the distance from the anterior superior iliac spine to the navel C> Lightly tap their facial nerve. If any twitching occurs, the test is positive D> Place a hand below the client's costal margin on the right side at the midclavicular line. Ask the client to breathe in. If client cannot breathe in due to pain, the test is considered positive

D> Place a hand below the client's costal margin on the right side at the midclavicular line. Ask the client to breathe in. If client cannot breathe in due to pain, the test is considered positive Being unable to breathe in due to pain when this action is performed reveals a positive Murphy's sign.

A nurse who is caring for an adult client with urinary incontinence is concerned about the possible consequences of overflow urinary incontinence. Which of the following considerations should the nurse utilize when managing this type of incontinence? Select all that apply. A. Teach the client how to perform pelvic floor exercises B. Initiate a referral to a genitourinary surgeon C. Contact the provider about checking a post-void residual D. Encourage increased fluid intake E. Allow plenty of time to void

E. Allow plenty of time to void--The nurse can help the client who has overflow incontinence by allowing plenty of time to void. A. Teach the client how to perform pelvic floor exercises---Pelvic floor exercises are important with some types of urinary incontinence. C. Contact the provider about checking a post-void residual---Overflow incontinence occurs when the client is unable to adequately empty his bladder and urine leakage occurs as a result. The nurse can ask about arranging a post-void residual, which is a test that can determine how much urine is left in the bladder after voiding.

A client who is recovering from surgery has been ordered to have a full liquid diet. Which foods can the client eat on this diet? Select all that apply. A. Cottage cheese B. Strained soup C. Pudding D. Runny, scrambled eggs E. Ice cream

E. Ice cream--Foods that are opaque liquid, or liquid at body temperature are part of a full liquid diet. Ice cream, sherbet and milk are included. B. Strained soup--Strained soup is normally liquid, so it is part of the full liquid diet. C. Pudding--A full liquid diet is appropriate for a client who is advancing their diet following a procedure or an illness that affects digestion. This diet includes opaque liquid foods and foods that are liquid at body temperature. This is the next advanced diet after a clear liquid diet. Examples include plain ice cream, sherbet, milk, pudding, custard, strained soups, strained vegetable juices and refined cooked cereal. This diet is deficient in many nutrients and does not typically provide enough calories, so the hope is that the client advances to the next diet stage, mechanical soft, in a timely manner.

A client is being treated for syphilis and the nurse knows that which of the following statements regarding tertiary syphilis infection are true? Select all that apply. Once in the tertiary stage, the client is no longer contagious Detecting and treating syphilis in the first two stages of infection prevents late-stage infection Tertiary syphilis can cause aortic aneurysm and seizures if untreated Intramuscular pencillin G halts the progression of tertiary syphillis Tertiary syphilis occurs within 10 years of primary syphilis infection

Once in the tertiary stage, the client is no longer contagious Tertiary infection is the only stage of syphilis that is not contagious. Detecting and treating syphilis in the first two stages of infection prevents late-stage infection Detecting syphilis early is the only way to prevent tertiary syphilis. Tertiary syphilis can cause aortic aneurysm and seizures if untreated Untreated tertiary syphilis can cause cardiac and neurological disasters, such as aortic aneurysm and seizure. Intramuscular pencillin G halts the progression of tertiary syphillis Penicillin infusion is the only way to treat tertiary syphilis and halt the progression of organ damage.

The nurse is caring for a client who is at risk for dumping syndrome after a partial gastrectomy. Nursing interventions to prevent this from happening include which of the following? Select all that apply. A. Increase consumption of fat and protein B. Lie down for 30 minutes after eating C. Choose whole grains over simple carbohydrates D. Eat larger meals less often E. Restrict fluids with meals

Restrict fluids with meals This is helpful to prevent dumping syndrome. Increase consumption of fat and protein Fat and protein is less likely to cause dumping syndrome than carbohydrates. Lie down for 30 minutes after eating Dumping syndrome occurs when the pyloric valve is bypassed following surgery, and the stomach rapidly pushes contents into the intestines. The contents then move rapidly through the bowels, which prevents nutrient absorption and causes diarrhea and pain. Restricting fluids until 30 minutes after meals, lying down for 30 minutes after eating, eating more protein and fat, and avoiding simple carbohydrates such as sugar and white flour are a few action to reduce the risk of dumping syndrome. Choose whole grains over simple carbohydrates Simple carbohydrates will move faster through the GI tract, and should be minimized.

A nurse is assessing a client and asking about symptoms. The nurse knows that which of the following symptoms would lead the nurse to suspect disseminated gonococcal infection (DGI)? A. Swollen joints B. Wart-like lesions on labia C. Diffuse rash D. Purulent vaginal discharge E. Fever

Swollen joints This occurs with DGI. DGI is known to cause arthritis as the infection causes inflammation at the joints. Purulent vaginal discharge This occurs with DGI. Purulent vaginal discharge occurs with DGI because the body is trying to rid the bacterial infection. Fever Fever occurs because gonorrhea is a bacterial infection that can cause a thermoregulatory response.


Conjuntos de estudio relacionados

Peds - Chapter 21: Nursing Care of the Child With a Genitourinary Disorder

View Set

Ch 16: Outcome Identification and Planning

View Set

OB PrepU: Chapter 20, OB PrepU: Chapter 19, OB PrepU: Chapter 16

View Set

Developing Cardiorespiratory Fitness

View Set

SEC+ 601. Chapter 14: Incident Response

View Set

Statistics for Behavioral Sciences: Chapter 9

View Set

PRACTICE QUIZ: CH.5 - Public Policing and Private Security

View Set

Chapter 10: Beginning and Ending the Speech

View Set

今天几月几号?What's The Date Today? (PinYin)

View Set

OSHA 30 - Quiz - 7 Material Storage

View Set